Case Files Psych Q/A

अब Quizwiz के साथ अपने होमवर्क और परीक्षाओं को एस करें!

49.3 A 38-year-old woman has adopted a new identity in a city 120 miles away from her hometown and has no memory of her prior life. Apparently, this event was precipitated by confrontation of her addiction to gambling and a threat of divorce. Which of the following could be a predisposing factor in her illness? A. History of head trauma B. History of an anxiety disorder C. Birth of a baby within 3 months D. Female gender E. Presence of a gambling addiction

A. A history of head trauma predisposes one to dissociative fugue.

1.3 Which of the following side effects common to SSRIs is the woman in Question 1.2 most likely to complain of in the future? A. Anorgasmia B. Headaches C. Insomnia D. Nausea E. Tremor

A. Although activation (causing insomnia), gastrointestinal symptoms (including nausea), and tremor are common side effects of SSRIs, only sexual dysfunction generally occurs later in the treatment course (after weeks to months).

56.1 Patients with which level(s) of mental retardation are capable of holding jobs? A. All levels. B. Mild only. C. Moderate and mild. D. Severe, moderate, and mild. E. Individuals with mental retardation are unable to hold jobs.

B. Although persons with all degrees of mental retardation can require some support to function in the community, those with mild mental retardation are able to hold a job. Individuals with moderate mental retardation are often able to manage small amounts of money and make change. Persons with severe and profound mental retardation have limited abilities to manage themselves and have difficulty learning these skills.

51.1 A 27-year-old female teacher with no psychiatric history comes in for treatment after a recent relationship with a boyfriend has failed; she states that she has had long-standing problems maintaining relationships with men and wishes to find "a solution." Since the breakup 4 weeks previously, she has had difficulty getting to sleep and frequently wakes up at 3 AM. She has no appetite and has lost 7 lb. She complains that her concentration in the classroom has deteriorated, that she has lost interest in her students, and that she often has crying spells during the day. She has withdrawn from friends and colleagues recently. Which one of the following diagnoses should the clinician consider first? A. Sleep disorder B. Major depressive episode C. Neurosis D. Bipolar disorder E. Borderline personality disorder

B. Although there can indeed be a neurotic conflict underlying the patient's difficulty with men, she currently has symptoms of major depression that must be addressed and treated before intensive dynamic psychotherapy or psychoanalysis could be considered. Her sleep problems and weight loss are likely part of her depressive illness and not likely to be a separate disorder. There is no evidence of a past or current manic episode consistent with bipolar disorder. She does not display the history of inappropriate anger, impulsivity, mood lability, or self-mutilation characteristic of borderline personality disorder.

32.3 The patient in Questions 32.1 and 32.2 feels that her headaches are now "unbearable." Which of the following treatments would be the most appropriate? A. Acetaminophen B. Biofeedback C. Lorazepam D. Nonsteroidal anti-inflammatory medication E. Oxycodone

B. Biofeedback and relaxation techniques have demonstrated efficacy in patients with pain disorder, particularly with headaches. Analgesics are often not helpful in these patients. Potentially addicting medications such as benzodiazepines and opiates should especially be avoided in these individuals given the chronic nature of this illness.

42.3 The patient's parents have been searching on the Internet for information about Tourette disorder and its treatment. They have a concern about tardive dyskinesia and would like to have the physician prescribe a medication which will minimize that risk. Which of the following medications, given the parents' concern, is the best choice for this patient? A. Pimozide B. Clonidine C. Risperidone (Risperdal) D. Haloperidol E. Clozapine (Clozaril)

B. Clonidine is an alpha-2 agonist that does not cause tardive dyskinesia as a side effect. It is moderately effective in the treatment of vocal and motor tics, although not as effective as some antipsychotics. All of the other agents are antipsychotic medications, and as such, have a risk of tardive dyskinesia associated with their use. Haloperidol, a typical antipsychotic, has the highest risk of tardive dyskinesia of the group, though tardive dyskinesia is a potential complication seen with all antipsychotic medications.

16.2 Which of the following complications would be most likely present in the patient in Question 16.1? A. Bradycardia B. Chest pain C. Delirium D. Hypothermia E. Respiratory depression

B. Cocaine intoxication causes numerous physical complications, including chest pain (from coronary vasospasm), tachycardia, diaphoresis, hypertension, and mydriasis. In severe overdose or when combined with other substances, it may cause seizures; however, delirium or respiratory depression are not commonly seen in intoxication.

44.2 Despite her protestations, the adolescent in the vignette above is diagnosed with anorexia. After stabilization of her nutritional status on a specialized inpatient unit, she is discharged home, with plans for follow-up therapy as an outpatient. Which of the following treatments have been shown to be effective in treating anorexia nervosa as an outpatient? A. Psychodynamic psychotherapy B. Family therapy C. Brief supportive therapy D. Group therapy E. Insight-oriented psychotherapy

B. Family therapy, both short-term and long-term, has been demonstrated to improve outcomes in adolescent patients with anorexia nervosa. Many of these family treatments are completed in stages, generally beginning with developing parental control over the eating and gradually turning this control back over to the adolescent with improvement in nutritional status. Some cognitive behavioral therapies have been shown to be effective but there is little evidence for the others listed.

55.2 Treatment of the case in Question 55.1 would likely involve administration of which of the following? A. Lorazepam B. Flumazenil C. Chlordiazepoxide D. Disulfiram E. Naltrexone

B. Flumazenil is a benzodiazepine receptor antagonist, which reverses the effects of benzodiazepines. Its use is restricted to use in the emergency department in instances of benzodiazepine overdose.

53.2 Gender identity disorder with a sexual attraction to males has been diagnosed in a 15-year-old boy sent to a psychiatrist. His parents are extremely unhappy with the boy's insistence on wearing women's clothes and want the psychiatrist to provide therapy "so that he won't think like this anymore." The boy is willing to talk to the psychiatrist but only if he can discuss the problems caused by the social ostracism he endures because of his wish to be a woman. Which of the following actions should the psychiatrist take? A. Inform the patient that his gender dysphoria will likely remit in time with psychotherapy alone. B. Inform the patient that psychotherapy has been found to be helpful but that sexual reassignment surgery is also an option once he is an adult. C. Inform the parents that their child will likely only experiment with homosexual behavior and will turn to heterosexuality once he is an adult. D. Inform the parents that they should forbid their son to wear women's clothes. E. Inform the patient's school that mechanisms must be put in place to reduce his social ostracism.

B. Gender identity disorders can be treated with psychotherapy and sex reassignment rehabilitation. Psychotherapy alone has not been found to be effective in relieving gender dysphoria and does not stop patients from continuing to be preoccupied with altering their sexual characteristics.

31.2 A 42-year-old woman is determined to "kick her heroin habit" at home without the use of methadone or any other prescription drug. Of the following over-the-counter medications, which is most likely to be of benefit to this patient as she goes through opioid withdrawal? A. Acetaminophen B. Ibuprofen C. Benadryl (diphenhydramine) D. Pseudoephedrine E. Dextromethorphan

B. Ibuprofen can help relieve the muscle cramps that are common to opioid withdrawal.

45.2 A 45-year-old man complains of lower back pain and weakness in his legs after lifting heavy boxes while at work. He says that he has not been able to go to work for several days. He requests treatment and a letter excusing him from work. On examination, he is found to have significant lumbar pain without spasms. The strength in his legs is decreased because of a lack of effort. His reflexes are within normal limits. A. Factitious disorder B. Malingering C. Conversion disorder D. Somatization disorder E. Hypochondriasis

B. In this case the most likely diagnosis is malingering. Although this man may indeed have some minor injury, his physical examination is remarkable only in revealing tenderness without spasms. His complaints of weakness and inability to work appear exaggerated given the lack of objective findings. The patient clearly has an obvious external motivation for embellishing his symptoms, namely, avoiding work.

45.4 A 50-year-old man is referred to a physician because he has ongoing migraine headaches. His headaches are chronic and bilateral, are worse with loud noises and light, and occur without aura or vomiting. His physical examination is unremarkable except that the patient does not appear to be in significant distress. When he is presented with various options for treatment, including nonsteroidal anti-inflammatory medications, he becomes angry, demanding that Tylenol with codeine is the only thing that has ever helped him. When he is told that non-narcotic medications should be tried first, he accuses the doctor of not believing him and storms out of the clinic. A. Factitious disorder B. Malingering C. Conversion disorder D. Somatization disorder E. Hypochondriasis

B. In this case the most likely diagnosis is malingering. This man presents with only subjective complaints; there are no significant medical findings or apparent suffering. He is angry and defensive, and he appears to be motivated solely by a desire to obtain narcotics rather than appropriate treatment.

44.3 Anorexia nervosa has been diagnosed in a 14-year-old girl, and she has been admitted to an inpatient psychiatric unit for treatment. On admission, she is found to be 5 ft 4 in tall and to weigh 82 lb. Over the next week in the hospital, she is weighed daily. She gains 4 lb the first day but subsequently loses 1 lb, and so at the end of the week she weighs 85 lb. It is discovered that the patient exercises all night long and hides all her food in napkins. What might account for the 4-lb weight gain early in the hospital stay? A. Early treatment motivation B. Excessive intake of water before the first weigh-in C. A decrease in metabolism, resulting in weight gain D. Scale error E. Patient binged on the first day but was unable to purge until after her first weigh-in

B. Patients admitted to psychiatric units for treatment of anorexia must be watched carefully because they go to great lengths to make it appear as if they are being compliant without really gaining any weight. Binging on water, placing heavy objects in their pockets before weighing, and other such maneuvers are often attempted.

57.4 A 32-year-old woman is admitted for second- and third-degree burns of her right hand which she attributes to accidentally spilling hot oil while she was cooking dinner. Upon evaluation, the surgeon recognizes the patient as a woman he had treated for a similar burn on the same hand 3 months ago. Further review of her medical records reveal that this is her sixth burn-related injury in 2 years. Which of the following is the most likely diagnosis? A. Conversion disorder B. Factitious disorder C. Malingering D. Hypochondriasis E. Body dysmorphic disorder

B. Patients with factitious disorders consciously fake symptoms to maintain the "sick role." In factitious disorder, there is not obvious secondary gain such as money or avoidance of work as seen in malingering.

33.4 Which of the following personality traits is most likely seen in this type of patient? A. Callousness B. Emotional lability C. Recklessness D. Cognitive dysregulation E. Grandiosity

B. Patients with histrionic personality disorder most often demonstrate emotional lability. Grandiosity is seen in antisocial personalities and narcissistic personalities; cognitive dysregulation is seen in borderline and schizotypal; recklessness in antisocial and borderline; and callousness in antisocial personality disorders.

59.1 A 19-year-old girl is brought to the emergency department by her friends, who are worried that she is not behaving normally. They suspect that she was experimenting with some type of drug, but are unsure what. Which of the following syndromes would be most consistent with amphetamine intoxication? A. Flushed face, slurred speech, unsteady gait B. Anorexia, diaphoresis, pupillary dilation C. Prominent hallucinations, pupillary dilation, incoordination D. Miosis, slurred speech, drowsiness E. Hyperphagia, conjunctival injection, tachycardia

B. Symptoms of amphetamine intoxication include anorexia, tachycardia, hypertension, pupillary dilation, and diaphoresis. A = alcohol intoxication, C = hallucinogen intoxication, D = opioid intoxication, E = cannabis intoxication.

13.3 The patient in Questions 13.1 and 13.2 is subsequently started on fluoxetine. Which potential side effect is classified by the Food and Drug Administration (FDA) as a "black box warning" issue? A. Neuroleptic malignant syndrome B. Suicidal ideations C. Tardive dyskinesia D. Serotonin syndrome E. Stevens-Johnson syndrome

B. The FDA has required a special warning, referred to as the "black box warning," be applied to use of antidepressants in children and adolescents. The FDA "black box warning" is the most serious type of warning in pharmaceutical labeling. This warning reads: "indicating that antidepressants may increase the risk of suicidal thinking and behavior in some children and adolescents."

47.1 A 29-year-old man is sent to a counselor at his employee assistance program, upon the urging of his supervisor. The patient had been working the night shift, but was recently promoted to a position on the day shift, with new supervisory responsibilities. Subsequently, his job performance has dropped off significantly. The patient states that since his transfer, he has been so nervous at work that he has not been able to think straight. He reports that his mood at home has been good, but that he knows he will fail at the new job because "I have always been such a dope when it comes to working with other people." After several sessions, the counselor diagnoses the patient with avoidant personality disorder. Which of the following would be the most helpful in assisting the patient to manage his anxiety in regard to his new job? A. Tell the patient that he needs to be more confident in his skills during this transition and "suck it up." B. Engage the patient in cognitive therapy to help him deal with his distorted thinking. C. Give the patient a beta-blocker to help him control his anxiety. D. Prescribe a benzodiazepine. E. Tell the patient that he is probably not ready for this job if he is this anxious.

B. The goal of cognitive-behavioral psychotherapy in such cases is to help patients critically examine if their assumptions about themselves and other people are correct.

6.1 A 48-year-old woman presents to a psychotherapist. The patient lives a very secluded life, largely consumed by working nights as a security guard and taking care of her elderly mother. She complains of feeling lonely, and is aware that she has a great deal of difficulty relating to other people. Which of the following conditions would most distinguish her issues from a person with schizoid personality disorder? A. Family history of a cousin with schizophrenia. B. A desire to engage in interpersonal relationships. C. Lack of hallucinations or delusional thinking. D. Her gender. E. A history of abstinence from alcohol.

B. The hallmark of schizoid personality disorder is a detachment and disinterest in social relationships. This patient is clearly distressed by her lack of social relationships.

41.4 The patient in Question 41.3 has now been seeing a therapist twice weekly for the last year. The therapist and the patient have a good working alliance. During one therapy session, the therapist comes to the session 4 minutes late. He apologizes to the patient, stating that he had an emergency involving another patient. During the session, the patient notes that the therapist "isn't as sharp as some of the therapists I hear on the talk shows." Which of the following defense mechanisms is the patient using? A. Denial B. Devaluation C. Isolation of affect D. Rationalization E. Splitting

B. The patient defends against his feelings of hurt and anger toward the therapist by using devaluation. Devaluation along with idealization and denial are considered primitive (lower-functioning) defense mechanisms used by patients with personality disorders such as narcissistic and borderline.

40.4 A 32-year-old single male patient is admitted with the provisional diagnosis of psychotic disorder, not otherwise specified, rule-out bipolar disorder. After 10 days, he is finally stabilized on valproic acid 2000 mg daily and aripiprazole 30 mg daily. The nurses are concerned his medications need to be increased or switched as he has been recently sleeping less and more agitated, often pacing the hallways. Upon examination, he admits to feeling "edgy," but he denies racing thoughts, increased energy, paranoia, or delusions. He states, "I just can't stop walking; I feel like I'm going crazy." A. Acute dystonic reaction B. Akathisia C. Neuroleptic malignant syndrome D. Parkinsonism E. Tardive dyskinesia

B. The patient is a young male with psychotic symptoms, rule-out bipolar disorder, stabilized on valproic acid and aripiprazole, but who recently has had worsening insomnia, anxiety, and restlessness (pacing). Given the improvement in his psychiatric symptoms, his current complaints are likely due to akathisia, a feeling of restlessness or anxiety, usually arising several weeks after treatment with antipsychotic medications. Consideration should be given to either decreasing his antipsychotic dose or adding another medication such as a beta-blocker or benzodiazepine.

58.2 Which of the following is a poor prognostic indicator in the treatment of fetishists? A. A stable adult relationship B. Presence of another paraphilia C. Normal intelligence D. Self-referral for treatment E. History of sexual relations without the paraphilia

B. The presence of multiple paraphilias is a poor prognostic indicator. Self-reporting is considered indicative of good prognosis as often the legal system is involved; a stable long-term relationship is a very good sign, as is achievement of sexual gratification without paraphilias.

56.3 An 18-year-old boy has benefited from training in social and occupational skills but has been unable to progress beyond the second-grade level in academic subjects. He needs supervision and guidance when under mild social or economic stress. Which level of mental retardation is being described? A. Mild B. Moderate C. Severe D. Profound E. Borderline level of intellectual functioning

B. This description refers to a person with moderate mental retardation. Refer to Table 56-2 for descriptions of the various degrees and functionality of mental retardation.

49.4 A 34-year-old woman is puzzled by a series of odd events. Over the past several months she has received numerous messages on her voicemail from men she does not know but who appear to know her. She has found items in her closet she does not remember buying, and has also began receiving bills in the mail for credit cards she does not remember opening. A friend recently asked her to show her pictures from a trip she took to Las Vegas, but the woman does not remember ever being in Vegas. What is the most likely diagnosis? A. Dissociative fugue B. Dissociative identity disorder C. Factitious disorder D. Malingering E. Borderline personality disorder

B. This patient is presenting with classic symptoms of dissociative identity disorder. Dissociative fugue would result in travel far away from home. Factitious disorder is the conscious production of symptoms for primary gain. Malingering is the conscious production of symptoms for secondary gain. While patients with borderline personality disorder may dissociate, the history presented here is not consistent with short periods of dissociation in conjunction with other features of that personality disorder.

37.3 A patient comes to her physician stating that for the last 6 months, since she started a new job, she has difficulty getting up in time for work. She notes that she is not tired around bedtime, and so she stays up for several hours playing computer games. When she finally does go to sleep, she has time to sleep for only 4 to 5 hours before she has to get up to go to work. She then finds herself groggy in the morning and fatigued throughout the day. This problem is interfering with her work at her job and thus is causing her distress. Prior to starting her new office job, the patient worked evening hours as a bartender and did not have a problem with sleeping. She takes no medications and uses no substances that could explain her sleep problems. The results of her physical examination are normal. Which of the following is the most likely diagnosis for this patient? A. Breathing-related sleep disorder B. Circadian rhythm sleep disorder C. Primary hypersomnia D. Primary insomnia E. Narcolepsy

B. This patient is suffering from a delayed-sleep-phase type of circadian rhythm sleep disorder. Circadian rhythm sleep disorder is characterized by a recurrent pattern of sleep disruption leading to excessive sleepiness and/or insomnia because of the mismatch between the sleep-wake schedule required in a person's environment (in this case, the demands of the patient's new job) and her circadian sleep-wake pattern. The sleep disorder must cause distress and must not be caused by a substance, a physical condition, or another mental disorder.

54.2 What is the best treatment option for the patient in Question 54.1? A. Antianxiety medication. B. Antipsychotic medication. C. Hospitalization. D. Psychotherapy. E. Reassure the patient that he is safe.

B. This patient would benefit from small dosage, short-term antipsychotic therapy to manage his delusional thinking. With a paranoid delusion, which is, by definition, a psychotic symptom, antipsychotic drugs are the therapy of choice. Since the patient is not exhibiting dangerous behavior to himself or others, hospitalization is not required. Reassurance, by definition of a delusion (fixed, false belief), will be unhelpful. Antianxiety medication is unhelpful and ineffective with a psychotic disorder, as is psychotherapy.

49.2 Dissociative fugue is distinguished from dissociative amnesia by which of the following? A. The presence of retrograde amnesia B. Travel far from home or family C. A precipitating traumatic event D. Creation of multiple identities E. Loss of consciousness

B. Travel far from one's home or family distinguishes dissociative fugue from dissociative amnesia; both are precipitated by trauma and are characterized by retrograde amnesia. In neither dissociative fugue nor dissociative amnesia are multiple identities created, as they are in dissociative identity disorder.

56.2 Which of the following is the most common cause of mental retardation? A. Fragile X syndrome B. Genetic deficits C. Idiopathic or unknown D. In utero exposure to toxins E. Lead intoxication

C. Although each of the conditions listed is associated with mental retardation, the largest percentage continues to be attributed to idiopathic causes.

39.1 A 29-year-old married woman with a prior history of major depression, with postpartum onset, in remission, has given birth to her second child 5 days ago. She describes intense but transient periods of sadness with crying spells since the delivery. While she has been able to enjoy moments with her infant daughter, she worries that she will be "a bad mother." Her sleep is disrupted with resultant fatigue, but her appetite and concentration are adequate. Although she is concerned that her depression will result in harm to her child, she denies any homicidal ideation. She also denies suicidal ideation, paranoia, delusions, or hallucinations. She has no medical problems and is not taking any medications except prenatal vitamins. Which of the following is her most likely diagnosis? A. Bipolar disorder B. Major depressive disorder, recurrent, with postpartum onset C. Postpartum blues D. Postpartum psychosis E. Cyclothymia

C. Although the patient has a history of postpartum depression (a significant risk factor for future episodes) and is suffering from crying spells, fatigue, and guilt, her likely diagnosis is postpartum blues. Postpartum blues usually peaks within the first week after delivery and resolves by 2 weeks. There is no evidence of a past or current manic episode (consistent with bipolar disorder) or psychotic symptomatology (consistent with postpartum psychosis). While there should be a high suspicion for developing a recurrence of her major depression, this patient does not have many of the associated neurovegetative symptoms, such as anhedonia, appetite change, difficulty concentrating, or suicidal ideation. In addition, major depressive episodes with postpartum onset typically occur 1 to 6 months postpartum.

35.4 Which of the following is the most useful approach for patients with factitious disorder? A. Confronting their feigning of symptoms B. Discharging them from the hospital C. Establishing a therapeutic alliance D. Pharmacotherapy E. Referring them to legal authorities

C. Although there is no specific treatment for factitious disorder, the best way to help these patients is to attempt to establish a therapeutic alliance and a working relationship. Although this can be difficult, only then can the patient's compulsion to feign illness be addressed and dealt with in a psychotherapeutic environment. Confrontation is necessary in some circumstances, but if an accusatory or a judgmental manner is employed, patients flee care and begin the cycle again at another hospital. The premature discharge of such patients from the hospital or referral to legal services has the same result, although in cases of factitious disorder by proxy (where a caretaker simulates illness in a child), referral to child protective services is necessary because this behavior is considered a form of child abuse. Pharmacotherapy use should be limited unless the patient has a comorbid axis I disorder. Due to abuse potential, medications should be used with caution.

58.4 A 55-year-old man complains of inability to achieve an erection. He has been worried about his health recently and takes antihypertensive medication. Which of the following would most likely differentiate between an organic and psychiatric condition? A. A lower-extremity myographic examination B. Magnetic resonance imaging of the lumbosacral spine C. An erection on awakening in the morning D. The interpretation of projective tests E. An electroencephalographic reading

C. An erection on awakening is good evidence of a nonorganic etiology. Because the imaging of the lumbosacral spine may or may not indicate the capacity for erection, these options are not as predictive.

38.2 A 35-year-old woman with somatization disorder comes to see a new doctor. Prior to seeing this physician, she obtained complete evaluations from at least four clinics. The most important part of the treatment plan for this patient is which of the following? A. A trial of analgesics. B. Antidepressant medication. C. Establishing a schedule for regular visits. D. Explain symptoms are due to psychological factors. E. Intensive psychodynamic psychotherapy.

C. Analgesics are generally not helpful in treating these patients, and they probably cannot be relied on to take psychotropic medications. Patients with somatization disorder have difficulty recognizing affects and so are not candidates for dynamic therapy, especially early in their treatment. The most important treatment intervention is establishing a relationship with one clinician and scheduling brief, regular visits. Increasing the patient's awareness of possible psychological factors involved in the symptoms is a long-range goal and should be done when and if the patient is willing to see a mental health professional.

41.1 A 22-year-old, single graduate student with narcissistic personality disorder is admitted to a hospital after a car accident in which his right femur is fractured. A medical student has been assigned to follow the patient, but when she enters the room and introduces herself as a medical student, the patient states, "Oh, I wouldn't let a medical student touch me—I need someone with much more experience than you." Which of the following statements by the medical student is most likely to lead to a successful interview with this patient? A. I know this will be boring for you, but it's just one of the things that you will have to put up with in the hospital. B. I know you must be scared to be in the hospital, but you will be safe here. C. I'm told that you are a very articulate person, and so I'm hoping you'll teach me what I need to know. D. I understand that you think you deserve only the best, but I have been assigned to you. E. Please don't make this difficult, I have to interview you as part of my job.

C. Appealing to the patient's narcissism by being admiring most often deescalates the patient as well as improves the therapeutic alliance in these cases.

35.2 Which type of personality disorder is most likely to occur comorbidly with factitious disorder? A. Antisocial B. Avoidant C. Borderline D. Obsessive-compulsive E. Schizoid

C. Borderline personality disorder is not uncommon in patients with factitious disorder. Individuals with either of these disorders often have similar histories of abuse, molestation, and emotional neglect. Patients with borderline personality disorder also act out their internal psychological conflicts on an interpersonal level, and they display the chaotic, labile affective state seen in factitious disorder.

46.4 The patient described in Question 46.2 can best be distinguished from Rett disorder because of which of the following? A. Social difficulties B. Lack of language development C. Absence of a period of normal development D. Evidence of mental retardation E. Sex of the child

C. Both syndromes show evidence of presence prior to the age of 36 months; however, in Rett disorder, the child has an early phase of normal development then gradually loses those skills toward further decline.

55.3 The acute crisis of the woman in Question 55.1 is averted. She is subsequently hospitalized for detoxification. Which of the following benzodiazepines is most likely to cause a withdrawal syndrome? A. Chlordiazepoxide B. Chlorazepate C. Alprazolam D. Diazepam E. Lorazepam

C. Generally speaking, shorter-acting agents are more likely to cause a withdrawal syndrome. Alprazolam is the shortest-acting benzodiazepine listed.

57.1 A 17-year-old boy presents with a complaint of his "legs giving out" for 1 week. During each episode he experiences a generalized painful sensation and within a few seconds he becomes weak and unable to move his arms and legs. The episodes last a few minutes. The patient is in the 11th grade earning Bs and Cs. His parents have recently separated after a long period of verbal abuse toward each other. His physical examination and neurologic workup are unremarkable. Laboratory studies are normal. Which of the following characteristics most distinguishes his diagnosis from other somatoform disorders? A. Symptoms are not fully explained by a medical cause. B. Symptoms are not intentionally produced. C. Symptoms involve only motor or sensory neurologic deficits. D. Psychological factors are related to symptom production. E. Secondary gain is involved.

C. In all somatoform disorders, the symptoms, pain, deficits, or preoccupation with a serious illness are not fully explained by a medical condition or are out of proportion. None of these symptoms are intentionally produced, as in factitious disorder or malingering. Psychological factors (conflicts or stress) are associated with the creation, exacerbation, and/or maintenance of the symptoms in all these disorders. Although somatization disorder involves a pseudoneurologic symptom, multiple, additional symptoms are also required. Conversion disorder is distinguished by having only a motor or sensory deficit.

60.2 The patient in Question 60.1 complains to her psychiatrist that all the nurses on the floor don't know what they are doing and they are rude. She later tells her nurse she is the best nurse on the floor and wished her psychiatrist cared about her the way the nurse did. Which of the following defense mechanisms is this patient using? A. Altruism B. Intellectualization C. Splitting D. Sublimation E. Undoing

C. Individuals with BPD often use the defense mechanism of splitting. Splitting is literally the "splitting off" a person's good and bad characteristics into two separate (and nonoverlapping) views of a person, which then alternate. Most healthy adults will be able to characterize people with their good and bad points at the same time. Borderline patients will characterize another as either all good, or all bad, depending on which side of the split the patient is currently seeing.

38.1 A 28-year-old woman comes to your clinic after being seen by her primary care doctor and several specialists. She complains that for the last 2 years she has experienced headaches, back and joint pain, abdominal pain with nausea and bloating, numbness and tingling in her upper extremities, and irregular menses. No physical cause for her symptoms can be found. The patient is insistent that she just can't work anymore due to "her pain" and wants you to sign a sick leave request form. Which of the following might help rule out somatization disorder? A. Age of this patient. B. Presence of four pain symptoms. C. Presence of secondary gain. D. Symptoms can't be explained by a medical condition. E. Symptoms lasting for 2 years.

C. Intentionally produced symptoms rule out somatization disorder. When external (secondary) gains such as avoiding work are present, malingering should be considered, and if malingering is indeed the diagnosis, it is likely that the symptoms are being produced intentionally. The other signs/symptoms on this list are consonant with a somatization disorder, and thus would not help rule it out.

29.3 Which of the following laboratory abnormalities would most likely be found in the patient in Question 29.2? A. Hypermagnesemia B. Hypoamylasemia C. Hypochloremic-hypokalemic alkalosis D. Elevated thyroid indices E. Hypercholesterolemia

C. Laboratory abnormalities found in individuals with bulimia nervosa demonstrate hypochloremic-hypokalemic alkalosis resulting from repetitive emesis. Hyperamylasemia and hypomagnesemia are also not uncommonly seen in such patients. Various electrolyte imbalances can occur as a result of frequent laxative abuse. Thyroid abnormalities are not common in individuals with bulimia nervosa.

52.3 An elderly woman presents to the emergency department due to a hip fracture. She reports that she "hasn't been feeling very well," recently, and is vague and hard to pin down regarding details. You think that there might be the odor of alcohol on her breath, and suspect alcohol abuse versus dependence. Which of the following findings would be most supportive of your concern? A. A healed scar from a previous fall several years ago B. Microcytic anemia C. Elevated gamma-glutamyl transpeptidase D. Slightly elevated aspartate aminotransferase (AST), with normal alanine aminotransferase (ALT) E. Mini-Mental State Examination score of 28/30

C. Laboratory tests would commonly show macrocytic anemia, elevated liver transaminase levels—particularly gamma-glutamyl transpeptidase in alcoholics.

46.1 Which of the following factors is most likely associated with a more positive prognosis for autistic disorder? A. Physical development/performance IQ B. Family socioeconomic status C. Language development/verbal IQ D. Presence of a seizure disorder E. Sibling order

C. One of the best predictors of the ability to improve the development of autistic children is the extent to which they have, or have begun to develop, language skills. This ability is often measured in verbal IQ testing.

47.3 Which statement best characterizes the difference between patients with avoidant personality disorders and those with schizoid personality disorders? A. Patients with avoidant personality disorders have fewer friends than those with schizoid personality disorders. B. Patients with avoidant personality disorders have higher self-esteem than those with schizoid personality disorders. C. Patients with avoidant personality disorders would like to have friends more than patients with schizoid personality disorders. D. Patients with avoidant personality disorders are better at accepting criticism than patients with schizoid personality disorders. E. Patients with avoidant personality disorders are less anxious than are patients with schizoid personality disorders.

C. Patients with avoidant personality disorders would desperately like to have social relationships, but they are afraid of criticism and/or rejection.

30.2 Treatment of ASD should focus primarily on which of the following? A. Biofeedback B. Debriefing the individual about the even C. Mobilizing social supports D. Pharmacologic treatments, such as selective serotonin reuptake inhibitors E. Psychotherapy

C. Pharmacologic treatments, psychotherapy, and biofeedback are primarily interventions for PTSD; the results of research are currently unclear regarding the benefits of debriefing. Mobilizing social supports is the most effective intervention in treating patients with ASD.

51.2 A 24-year-old male engineering student comes to the counseling center because of "problems with sex." He is heterosexual and able to engage in casual relationships. While he experiences normal sexual excitement, when the opportunity for intercourse occurs, he becomes anxious and cannot maintain an erection. He can masturbate without difficulty, and his family physician has told him that he is "completely normal physically." Upon further history, the patient states that his father was a military officer who was absent for significant periods of time while the patient was in his early school years; the patient was an only child who was very close to his mother. He denies pervasive depression and has no psychiatric history. Which of the following is the most appropriate treatment for this patient? A. Cognitive-behavioral therapy (CBT) B. Antidepressant medication C. Psychodynamic psychotherapy D. Interpersonal therapy E. Dialectical behavioral therapy

C. Psychodynamic psychotherapy is indicated for this young man, who has a circumscribed conflict about performing sexually. Although additional history is needed, there is a suggestion that early conflicts about sexuality arose because his father was absent from the home at a time when normal children entertain fantasies about marrying the parent of the opposite sex. As the patient is not depressed, he does not require antidepressant treatment. Neither CBT nor interpersonal therapy is likely to elicit the unconscious conflict troubling him. Dialectical behavioral therapy is a form of CBT developed for the treatment of borderline personality disorder—there is no evidence for this diagnosis in this patient.

43.3 Generalized anxiety disorder secondary to a medical condition is diagnosed in a 23-year-old woman after she began having obsessions and compulsions after falling from a horse. Although her head trauma was treated, and she apparently suffered no sequelae, her obsessions and compulsions have continued. If an SSRI is given, which of the following medication side effects will most likely develop? A. Orthostatic hypotension B. An increase in the QT interval C. Anorgasmia D. Tardive dyskinesia E. Seizures

C. Selective serotonin uptake inhibitors are the treatment of choice for obsessions and compulsions, and the most common side effect of these agents is sexual dysfunction, primarily anorgasmia in women and delayed ejaculation in men.

36.3 The pediatrician orders a sleep study on this patient which documents the presence of sleep-disordered breathing, a commonly concurrent phenomenon with sleep terrors. What treatment for the sleep terror might best be considered at this point? A. A selective serotonin reuptake inhibitor B. A benzodiazepine sleeping agent C. Adenoidectomy or tonsillectomy D. Reassuring the parents that the patient will not harm herself E. Restraining the patient in bed

C. The best treatment for night terrors with the documented presence of another sleep disorder is definitive treatment of that sleep disorder. In this case, an adenoidectomy or tonsillectomy should be considered in consultation with an ear, nose, and throat specialist. Nearly all cases of sleep terrors resolve if the primary sleep disorder is addressed appropriately.

42.4 A patient with ADHD is treated with methylphenidate during the school year. After several months of treatment, his teachers and parents note that he has developed both motor and vocal tics. What should be the first course of action for these symptoms? A. Begin treatment with haloperidol. B. Discontinue the use of methylphenidate. C. Switch medication to atomoxetine. D. Reduce the dose of methylphenidate. E. Administer an anticonvulsant.

C. The development of tics as a side effect of stimulant medication is relatively common. While these tics diminish in severity or cease when the dose is reduced, reduction of dose often results in an increase of ADHD symptoms. Because atomoxetine is quite effective for treating ADHD without stimulating tics, a trial on atomoxetine should be considered. If ADHD symptoms and/or tics continue on atomoxetine then addition of clonidine or guanfacine should be considered.

34.1 Adjustment disorder is diagnosed in a 45-year-old woman who was fired from a job she held for 20 years. She undergoes supportive psychotherapy. Nine months later, she is seen by her physician, but none of her symptoms have resolved. During this time, she has found another job that is similar to her first position in duties and salary. Which of the following is the most likely diagnosis? A. Adjustment disorder B. Posttraumatic stress disorder C. Major depressive disorder D. Bipolar disorder E. Schizoaffective disorder

C. The duration requirement for symptoms occurring after the stressor resolved is met for a major depression. Since the stressor has been removed, an ongoing adjustment disorder would be an incorrect answer. There is no evidence for psychosis or manic moods, so options relating to bipolar or schizoaffective disorder would be incorrect. PTSD would not be a viable option because the patient has not had a life-threatening stressor occur.

45.1 A 23-year-old pregnant woman complains of an inability to feel her legs. She wonders if the fetus is grabbing her spinal cord. Although she does not appear concerned about her condition, on further questioning she admits that her pregnancy was unplanned and that it has been a source of stress for her and her husband. Her neurologic examination is unremarkable except for decreased sensation below her waist. The results of a computed tomography scan and magnetic resonance imaging of her brain and spine are normal. A. Factitious disorder B. Malingering C. Conversion disorder D. Somatization disorder E. Hypochondriasis

C. The most likely diagnosis for this woman is conversion disorder. She presents with symptoms of a neurologic disorder without an obvious cause or trauma. She does not appear particularly concerned about her symptoms (la belle indifference), and there is no obvious possibility of obtaining external gain. Her motivation does not seem to be assuming the sick role but rather expressing an unconscious conflict involving her unwanted pregnancy.

50.1 A 35-year-old man is brought to a psychiatrist's office by his wife. He had previously suffered a major depressive episode 2 years prior, and ceased medications 6 months ago. More recently, the patient had been working many overtime hours for several weeks to complete a project at work, and had slept much less than normal without apparent ill effect. When the project was completed, the patient continued to sleep little, and shifted his activities to socializing and drinking with his work colleagues. The patient admits he has not drunk this heavily since college. For the past few days, the patient has "crashed" back into depression. Which of the following is the most likely explanation for this patient's condition? A. Exacerbation of major depression B. Substance-induced mood disorder (alcohol) C. Bipolar disorder D. Adjustment disorder E. Circadian rhythm sleep disorder

C. The patient describes a pattern of decreased need for sleep, yet with no decrease in energy level, which is highly suggestive of a diagnosis of bipolar disorder. He may have had similar episodes in the past, but such patients generally seek help when they are depressed, as opposed to hypomanic or manic. Increased goal-directed activity and excessive pleasure-seeking activities such as drug use are hallmarks of a manic episode. However, it is often very difficult to distinguish between a primary mood disorder and a substance-induced one, without an extended period of abstinence accompanied by continuing mood complaints.

58.1 A married pharmacist comes in for treatment at the insistence of his wife, who was disturbed to find that he was wearing some of her undergarments under his clothes. He admitted to her that he often masturbates when wearing her underwear and fantasizes about wearing it while having intercourse with her. Which of the following words best define this paraphilia? A. Exhibitionism B. Fetishism C. Transvestism D. Voyeurism E. Masochism

C. The patient is a transvestite because he wears the articles of a woman's clothing in order to achieve sexual excitement. Transvestism is a paraphilia in which the individual seeks sexual gratification primarily through the use of objects which are closely associated with the body, such as shoes or underwear.

46.2 A 4-year-old boy who is an only child starts preschool. His parents are quite nervous about this and describe themselves as protective and over-involved. However, they are looking forward to seeing him more involved with school, as he has had little peer interaction prior to this because he is an only child, and he has never shown much of an interest in interacting with others. He has never used many words which the parents attribute to his isolation as an only child. They tell the teacher he has always done best with a strict schedule and doesn't tolerate changes well. He has always seemed to have a very narrow repertory of play—focusing primarily on spinning objects such as tops and balls. He comes to the classroom for the first time and runs right to these toys and does not say goodbye to his mother, nor even acknowledge she is leaving. Other children attempt to play with him but his response is to either ignore them or get angry at their advances. This last response in someone with autism might best be described as which of the following? A. Rigidity B. Stereotyped behavior C. Lack of social reciprocity D. Poor language development E. Obsessional thinking

C. The patient's lack of response and then inappropriate response to the social advances of his peers might best be considered a symptom of impairment of social reciprocity. Social reciprocity involves the appropriate sensing and then interpretation of verbal and nonverbal social cues that guide our interactions.

35.1 Which of the following is most likely the motivation behind the behavior displayed in factitious disorder? A. The motivation is unconscious and thus the patient is unaware of it. B. Desire to avoid jail. C. Desire to take on the patient role. D. Desire to obtain compensation. E. Desire to obtain narcotics.

C. The primary desire in factitious disorder is to assume the sick role and be taken care of. In contrast, in malingering, the motivation is to achieve a tangible gain (such as avoiding work, school, or a prison sentence) or to obtain narcotics or financial compensation. If the motivation is unconscious, conversion and other somatoform disorders would need to be considered.

59.2 In the previous case, urine toxicology confirms intoxication with amphetamines. Which of the following withdrawal syndromes would be expected? A. Diarrhea, piloerection, yawning B. Delirium, autonomic hyperactivity, visual or tactile hallucinations C. "Crash" of mood into depression, lethargy, increased appetite D. Tremor, headache, hypertension E. Postural hypotension, psychomotor agitation, insomnia

C. The withdrawal symptoms include depressed mood, lethargy, and increased appetite. A = opioid withdrawal, B = delirium tremens, D = alcohol withdrawal, E = sedative/hypnotic withdrawal.

40.2 A 27-year-old man is admitted for acute psychotic symptoms, consisting of command hallucinations to harm others, paranoid delusions, and agitation. He is begun on olanzapine 30 mg daily. After several days, he becomes calmer but more withdrawn. When approached by the nurses, he is found to be lying in bed, eyes open but not responsive. He is noted to be sweating but is resistant to being moved. His vital signs demonstrate a temperature of 101.4°F, blood pressure 182/98 mm Hg, pulse 104/min beats per minute (bpm), and respiration 22 breaths per minute. A. Acute dystonic reaction B. Akathisia C. Neuroleptic malignant syndrome D. Parkinsonism E. Tardive dyskinesia

C. This acutely psychotic patient has been started on an antipsychotic medication, namely olanzapine, and he has now developed acute mental status changes, diaphoresis, rigidity, and fluctuating vital signs. These are signs and symptoms consistent with neuroleptic malignant syndrome, a medical emergency. Antipsychotic medications should be discontinued immediately, and supportive measures need to be employed.

53.1 A 35-year-old man being seen for major depression shares that he enjoys dressing as a woman and masturbating in private. He finds cross-dressing very arousing sexually but is married and his wife has become aware of this. She is very upset and there have been marital problems over his behavior. At work and in other settings he functions in typical male roles and activities. He has had two sexual experiences with men before he got married. He feels very committed to his marriage and finds his wife sexually attractive. The best diagnosis for this patient would be which of the following? A. Mixed personality disorder with schizotypal and borderline features B. Gender identity disorder C. Transsexual fetishism D. No diagnosis as bisexuality is not a psychiatric disorder E. Body dysmorphic disorder

C. This individual has transvestic fetishism. He has a male gender identity and is comfortable with male role function. He cross-dresses for sexual excitement but it has a negative impact on his marriage. There is nothing presented to support a diagnosis of personality disorder, that is, he does not seem to have problems functioning at home or at work in his life in general. He also does not seem to have symptoms supportive of body dysmorphic disorder—he does not seem displeased or disgusted with his body shape or presence/appearance of sexual organs per se. While it is true that bisexuality and homosexuality are not psychiatric diagnoses, the transvestic fetishism is.

58.3 A 23-year-old man comes to his physician asking for sexual reassignment surgery. He states that for "as long as I can remember" he has felt that he was born in the wrong body. He states that he believes that "truly I am a woman" and is disgusted by his male body habitus. He wishes to have his penis removed, and would like female breasts and genitalia. He considers himself a heterosexual because he is attracted to men. Which of the following best describes this man? A. He has a paraphilia. B. He has fetishism. C. He is a transsexual. D. He is having a psychotic delusion. E. He is a sadist.

C. This man is a transsexual, an individual who feels as if he is trapped in a body of the wrong gender.

48.1 A 10-year-old boy presents with episodes of somatic complaints, anxiety, and crying at school which resolves when he is sent home. He won't go anywhere without his mother. Which of the interventions would be appropriate in the plan of treatment? A. Place on home-bound tutoring to be provided by the school district. B. Prescribe lorazepam prn for anxiety episodes. C. Place the patient on fluoxetine in low dose. D. Immediately restrict access to the mother until anxiety symptoms cease. E. Reassure the mother that the patient is going through "a phase" and that this will pass with little impact on youth's subsequent life.

C. This medication is one of several agents in the class collectively called SSRIs. Lorazepam is a benzodiazepine that is habit forming and more likely to disinhibit the child. Home-bound tutoring will only reinforce the patient's dependency on his mother. The separation of the patient should be gradually done in degrees. This disorder is not a phase but an indicator of subsequent risk for psychiatric illness as an adult.

33.3 A 20-year-old woman comes to see a psychiatrist at the insistence of her mother, who states that her daughter just "isn't herself." The patient has dressed in brightly colored clothes and worn large amounts of makeup for the past 3 weeks. She acts overtly seductive toward her colleagues at work, is more distractible, and is easily irritated. She also sleeps less, claiming that she "no longer needs it." Which of the following diagnoses best fits this patient's presentation? A. Histrionic personality disorder B. Borderline personality disorder C. Bipolar disorder, mania D. Narcissistic personality disorder E. Delusional disorder

C. This patient has a new onset of behavior that is unlike her usual personality. It includes dressing in loud clothing and wearing lots of makeup, as well as being seductive. She is distractible, irritable, and needs less sleep than usual. All these symptoms point to a manic episode (assuming neither a medical condition nor a substance can account for the sudden change in functioning).

54.1 A 36-year-old man comes to a physician's office with a chief complaint that "people are out to hurt me." Despite being reassured by his wife that this is untrue, the patient is convinced that men are observing his behavior and actions at home and at work, using telescopic lenses and taping devices. He has torn apart his office on more than one occasion looking for "bugs." The patient's wife says that this behavior is relatively new, appearing somewhat suddenly after the patient was robbed on the way to his car approximately 6 months previously. Which of the following symptoms best describes what the patient is experiencing? A. Ideas of reference B. Hallucinations C. Paranoid delusions D. Paranoid ideations E. Thought disorder

C. This patient's problem is more than mere suspiciousness; he has full-blown paranoid delusions: fixed, false beliefs. Paranoid ideation is mere suspiciousness—the worry that harm is meant by others. People with paranoid ideation may often be consoled or reassured by a trusted friend, and they do not often act on these suspicious. By contrast, people with paranoid delusions have fixed (ie, they are not able to be reassured) and false beliefs that others mean them harm. These patients may act on these beliefs as well, for example, this patient tearing apart his office looking for "bugs."

39.2 Which of the following treatment approaches is the most appropriate for the patient in Question 39.1? A. Antidepressant medication B. Antipsychotic medication C. Continued observation D. Electroconvulsive therapy E. Mood stabilizer

C. While this patient may develop a recurrence of major depression requiring antidepressant therapy, postpartum blues is self-limiting, not necessitating formal treatment. Continued observation and reassurance are all that are recommended. Postpartum psychosis requires treatment with an antipsychotic in combination with an antidepressant or a mood stabilizer, or a course of electroconvulsive therapy.

44.1 A 16-year-old girl is brought to a physician by her mother, who states that her daughter has been losing weight steadily. The adolescent denies there is a problem and states that she is in no way underweight. The physician determines that the girl is 5 ft 6 in tall and weighs 90 lb. Which of the following laboratory tests is most helpful in assessing the severity of starvation in this patient? A. Complete blood count and differential white blood cell count B. Thyroid function studies C. Serum potassium level D. Determination of albumin level E. Liver function studies

D. A determination of the albumin level can assist in assessing the current extent of starvation in a patient. It is an important index in the treatment of anorexic patients.

50.2 A 22-year-old woman presents to the emergency department with complaints of depression and suicidal ideation. She admits that up until 24 hours ago, she was heavily abusing cocaine. Which of the following findings would be most common with this presentation? A. Miosis, slurred speech, drowsiness B. Nystagmus, hypertension, muscle rigidity C. Conjunctival injection, increased appetite, dry mouth D. Fatigue, increased appetite, vivid and unpleasant dreams E. Mydriasis, gooseflesh, rhinorrhea, muscle aches

D. A is consistent with opioid intoxication. B describes phencyclidine intoxication. C is cannabis intoxication. E is consistent with opioid withdrawal.

53.3 A 29-year-old man with a diagnosis of gender identity disorder wishes to undergo sex reassignment surgery. Which of the following treatment steps is strongly related to a positive outcome of sex reassignment? A. Screening the patient for psychopathology B. Treating the patient with an antidepressant before surgery C. Treating the patient with hormones D. Real-life experience in the community E. Ongoing supportive psychotherapy

D. According to the minimum standards of the Harry Benjamin International Gender Dysphoria Association, the real-life experience of living in the community in the desired sex role for at least 3 months before hormonal reassignment, and 12 months before surgical reassignment, is recommended, since these experiences are correlated with a positive outcome.

52.4 A 63-year-old man presents to the emergency department with complaints of anxiety. He describes a long history of daily, heavy alcohol use, and 2 days ago "quit cold turkey." He appears visibly tremulous, flushed, and diaphoretic. His temperature, blood pressure, and pulse rate are elevated. The results of his physical examination are otherwise unremarkable, but his laboratory tests demonstrate low serum albumin and low protein levels, as well as an elevated prothrombin time/partial prothrombin time value. He is admitted to the medical service for alcohol detoxification. Which of the following medications would be most appropriate in treating this patient? A. Alprazolam B. Chlordiazepoxide C. Diazepam D. Lorazepam E. Clonazepam

D. Although all these medications are benzodiazepines, only lorazepam is metabolized solely by glucuronidation, which is not as dependent on liver functioning. The metabolism of the other benzodiazepines is much more dependent on liver function. In this patient (who has evidence of poor liver function), using high doses of medications that are dependent on liver function for their degradation could result in excessive drug levels in the blood of an overly sedated patient.

57.2 Which of the following would be the most effective approach for the patient in Question 57.1? A. Confrontation about intentionally producing symptoms B. Explaining that the symptoms are not real C. Reassurance that a neurologic cause will be found D. Suggestion that symptoms will improve with time E. Suggestion that the family begins therapy

D. Although the deficits often remit spontaneously, suggesting that they will improve can facilitate the process. These patients do not intentionally produce their symptoms, and explaining that their deficits are not real may aggravate the situation and worsen their problems. Whereas reassurance about their likely improvement is appropriate, implying that their symptoms are caused by a neurologic illness (assuming that this has been ruled out) may serve only to reinforce their defensive use of a physical illness to express their psychological problems.

55.1 A 55-year-old, unresponsive woman is brought to the emergency department after an apparent suicide attempt. Earlier that day, she refilled her monthly prescription for a benzodiazepine, which she had been prescribed for panic disorder. The empty pill bottle was found on her nightstand by the paramedics. Concurrent ingestion of which of the following substances is most likely to worsen the prognosis of her overdose? A. Cannabis B. Cocaine C. Citalopram D. Alcohol E. Lysergic acid diethylamide (LSD)

D. Benzodiazepines are rarely lethal in overdose by themselves. However, when taken with other sedative-hypnotic drugs, especially alcohol, the effects of their potentiation can be lethal.

59.3 A 20-year-old man is brought to a mental health center by his parents, who are at their wit's end due to their son's drug problem. The son is sullen and completely uncommunicative. The parents, who are extremely naïve about the world of street drugs can only guess by his behavior that he is abusing "uppers." Which of the following findings might help differentiate between the abuse of cocaine versus amphetamine? A. Rhinorrhea B. "Track marks" on his arms C. Severe smoker's cough and respiratory problems D. Extremely poor dentition E. Weight loss

D. Cocaine, amphetamines, and other stimulant drugs are quite similar in their presentation. Both cocaine and amphetamine may be used via smoking, insufflation (potentially leading to nasal problems), or intravenously. However, "meth mouth" is commonly seen with abuse of methamphetamine, caused by lowered saliva production in conjunction with cravings for sugar.

49.1 A man who appears to be approximately 70 years of age is brought to the emergency department by the police. He was picked up after he tried to order food in a restaurant but had no money to pay the bill. He is oriented to place and time and gives his name as "Bill," but he cannot remember where he lives, his telephone number, or the names of his family members. He does recall that he served in the Pacific during World War II and that he was raised in rural New Hampshire. The results of his physical examination are essentially normal, and his routine laboratory tests reveal mild anemia. Which of the following is the most likely diagnosis? A. Dissociative amnesia B. Dissociative fugue C. Alcohol dependence D. Dementia E. Dissociative identity disorder

D. Dementia. This patient has preserved some past memory, which is characteristic of dementia but not of dissociative fugue or amnesia.

53.4 Patients with schizophrenia or other psychotic disorders may present with delusional claims of cross-gender issues. Which of the following pieces of history would suggest a male patient's claim of cross-gender identity is due to delusions? A. The patient says he feels as if he is a member of the other sex but does not actually believe that he is a member of the other sex. B. He has felt that he was the wrong gender from a young age. C. He dresses in the attire of the other sex. D. He actually believes he is a member of the other sex.

D. Gender identity disorder begins at a young age and individuals with this disorder frequently cross dress. They realize what their physical assigned sex is but believe they feel that they are members of the opposite sex. Individuals with delusions actually believe they are a member of the opposite sex even without sex change surgery (a male who believes he is physically a female even with male anatomy).

59.4 A 38-year-old white man is brought into the emergency department by the police. Several officers are needed to control the patient, who is psychotic and extremely agitated, requiring placement in full-leather restraints. The officers had received a complaint that the patient attacked several individuals at a biker party for no apparent reason, although witnesses on the scene indicate that the patient had been smoking methamphetamine. Which of the following pharmacological interventions is the most appropriate? A. Citalopram B. Diazepam C. Ascorbic acid D. Haloperidol E. Bupropion

D. Haloperidol and antipsychotic medications are best suited to target psychosis and agitation. Diazepam would be preferable to address agitation in the absence of psychosis. Antidepressants such as citalopram and bupropion would not be useful acutely. Bupropion may be preferable to treat depression after acute withdrawal. The use of ascorbic acid to acidify the urine in PCP intoxication is no longer recommended.

33.2 A 23-year-old woman with a diagnosis of histrionic personality disorder comes to see her physician for the chief complaint of frequent headaches. As the (male) physician is taking the patient's history, he notices that she is frequently reaching across the desk to touch his arm as he talks to her, as well as leaning far forward in her seat to be nearer to him. Which of the following responses is the most appropriate from the physician? A. Tell the patient to stop touching him immediately. B. Move his seat further from the patient so that she cannot reach him. C. Tell the patient that she will be referred to a female physician. D. Tell the patient that he understands her fear about her headaches, but touching him is not appropriate. E. Tell the patient he understands her gratitude in this situation.

D. Histrionic patients often display inappropriate or seductive behavior. This is best managed by being tactful and sympathetic to the patient, but firmly and clearly placing boundaries on such behavior.

57.3 A 42-year-old man returns to his internist for the fourth time in 5 months with the same symptoms of intermittent numbness of his fingers and indigestion. Although his medical workup has been unremarkable, this has failed to reassure him. He is now concerned that he has celiac disease and requests a gastrointestinal (GI) consultation. Which of the following is the most likely diagnosis? A. Conversion disorder B. Factitious disorder C. Malingering D. Hypochondriasis E. Body dysmorphic disorder

D. Hypochondriasis is characterized as a person's preoccupation with the fear of contracting or the belief of having a serious disease. This fear stems from a misinterpretation of physical symptoms or functions. Unlike conversion disorder, hypochondriasis is not confined to one or more sensory or motor deficits. Since the patient is not consciously faking symptoms to maintain a sick role, this would not be considered a factitious disorder. If the patient were malingering there would be an obvious secondary gain. Patients with body dysmorphic disorder have a preoccupation with an imagined defect in appearance.

21.3 Despite a course of psychotherapy, the patient in Questions 21.1 and 21.2 continues to suffer from recurrent nightmares, flashbacks, hypervigilance, and emotional numbing. Which of the following medications is most likely to be helpful as monotherapy in this patient? A. Buspirone B. Risperidone C. Alprazolam D. Prazosin E. Valproic acid

D. Individuals with PTSD often respond to SSRIs or SNRIs. Additionally, alpha-1 antagonists such as prazosin have demonstrated efficacy in significantly reducing the symptom clusters of PTSD. However, buspirone, second-generation ("atypical") antipsychotics (such as risperidone), benzodiazepines, and mood stabilizers (such as valproic acid) are not recommended as monotherapy for the treatment of PTSD. Although alprazolam might assist in decreasing the patient's general anxiety, the incidence of substance abuse is high among patients with PTSD; thus, addictive medications should be avoided in these individuals.

6.3 A woman with schizoid personality disorder was involved in a motor vehicle accident in which she was rear-ended by another car. The driver of the other car refused to take responsibility for the accident and hired a lawyer to provide his defense. The woman spends hours each day thinking about the specifics of the accident, including such details as the color of the cars involved and what each party to the accident was wearing. Which of the following defense mechanisms, common to patients with schizoid personality disorder, is the woman using? A. Sublimation B. Undoing C. Projection D. Intellectualization E. Introjection

D. Intellectualization is characterized by rehashing events over and over.

36.1 A young child wakes up in the middle of the night screaming, frightened, and running to his parents room in this state. He is consolable and recalls a frightening dream in the morning at breakfast. In which stage of sleep did this nightmare likely occur? A. Stage 1 B. Stage 2 C. Stage 3 D. Rapid eye movement sleep E. Light sleep

D. Nightmare disorder is a parasomnia usually associated with the REM stage of sleep. Night terrors usually occur during non-REM sleep.

13.2 The patient in Question 13.1 begins treatment for his condition. As treatment develops, the parents inform you that he had an episode of glomerulonephritis with antibiotics several weeks ago prior to developing these symptoms. What other possible issue might be related to his current symptoms? A. Sydenham chorea B. Asperger syndrome C. Passive-aggressive personality disorder D. Pediatric autoimmune neuropsychiatric disorders associated with streptococcal infections E. Huntington disease

D. Pediatric autoimmune neuropsychiatric disorders associated with streptococcal infections, PANDAS, have been specifically associated with development or exacerbation of OCD in children and adolescents. The symptoms will develop temporally after a demonstrated streptococcal infection.

54.3 A 42-year-old woman undergoing psychotherapy storms into her therapist's office for her session and angrily accuses the therapist of "trying to undermine her intelligence." After a discussion with the therapist, it becomes clear that it is the patient who is second-guessing herself, thereby "undermining" her own intelligence. Which of the following defense mechanisms is this patient using? A. Denial B. Identification with the aggressor C. Intellectualization D. Projection E. Reaction formation

D. Projection is a defense mechanism by which individuals deal with conflict by falsely attributing to another their own unacceptable feelings, impulses, or thoughts. Blaming others for their own sentiments and actions directs the focus away from the person doing the accusing. For example, a patient who is angry with his therapist suddenly starts accusing the therapist of being angry with him.

33.1 A 35-year-old woman with histrionic personality disorder has seen her psychotherapist once a week for the past year. During a session, the therapist tells the patient that he is going to be on vacation the following 2 weeks. When he returns from the vacation, the patient tells him that she felt he abandoned her and says, "You didn't even bother to tell me that you would be away." This lapse in memory can best be described as which defense mechanism common to patients with histrionic personality disorder? A. Sublimation B. Splitting C. Undoing D. Repression E. Displacement

D. Repression is a common defense mechanism in patients with histrionic personality disorder.

38.4 A 29-year-old woman presents to establish care. She complains of a 6-month history of headaches, fatigue, lower extremity tingling, decreased libido, diffuse myalgias and arthralgias, and depression. She and her husband and recently separated and she has taken a leave of absence from work because of her physical problems. Three years ago she had an episode of "lightning pains" down her arms that resolved. Her past medical history is otherwise unremarkable and she has no history of psychiatric illnesses. She was adopted and has no knowledge of her family medical history. While talking to you, she becomes tearful. What is the most appropriate next step? A. Refer the patient for counseling. B. Start an antidepressant. C. Encourage her to see the connection between her physical symptoms and psychological distress. D. Complete a detailed history, physical examination, and order appropriate blood work. E. Schedule her for a return appointment within the next 2 weeks.

D. Somatization disorder generally presents with nonspecific symptoms. The most important initial step is to rule out medical causes.

37.2 You offer the patient in Question 37.1 a short course of lorazepam, but he declines. Which of the following should you next recommend to help his sleep? A. Eat a late evening meal. B. Exercise prior to bedtime. C. Sleep later on the weekends. D. Take a hot bath in the evening. E. Take naps during the day.

D. Taking a hot bath near bedtime is an effective technique for inducing sleep in some patients. The other options listed do not help and are actually likely to worsen insomnia.

48.3 When starting an SSRI, such as fluoxetine, in an adolescent patient with separation anxiety disorder, the FDA recommends the clinician monitor closely for which of the following? A. Hypovolemia B. Hypertension C. Anorexia D. Suicidal thoughts E. Delusions

D. The FDA recently placed a black box warning for the use of antidepressants in children and adolescents. This warning reminds clinicians of some evidence indicating a possible increased incidence of suicidal thoughts among adolescents using antidepressants—particularly SSRIs.

35.3 Which of the following scenarios is most consistent with factitious disorder? A. Feigning psychosis to avoid criminal charges B. Lying about back pain to receive time off from work C. Pseudoseizures in the context of a family conflict D. Placing feces in urine to receive treatment for a urinary tract infection E. Recurrent fears of having a serious illness

D. The hallmark of factitious disorder is intentional feigning of a physical or psychiatric illness in order to assume the sick role. Examples include injecting oneself with insulin to create hypoglycemia, taking anticoagulants to fake a bleeding disorder, and contaminating urine samples with feces to simulate a urinary tract infection. Lying about back pain in order to avoid work or feigning psychosis to avoid criminal charges is an example of malingering. Pseudoseizures are an example of a conversion disorder. Fear of having a serious disease caused by misinterpretation of bodily sensations is characteristic of hypochondriasis.

14.3 A 48-year-old woman is brought to the emergency department. She is unresponsive to questions, stumbles around the room, and is agitated. On physical examination, you notice that she smells of alcohol, and she is not cooperative during the remainder of the examination. Administration of what medicine would be the most appropriate initial treatment? A. Benzodiazepine B. Disulfiram C. Glucose D. Thiamine E. An antipsychotic agent

D. The most appropriate treatment is administration of thiamine. This patient presents with Wernicke encephalopathy, characterized by the triad of delirium, ataxia, and ophthalmoplegia. Thiamine must be given prior to glucose in patients suspected of having this disorder.

4.1 An 18-year-old man presents with 3 days of an irritable mood, decreased sleeping, talkativeness, increased energy, and distractibility. He has no personal or family psychiatric history and no current medical problems. His mental status examination is remarkable for psychomotor agitation and an irritable affect. He is paranoid but denies delusions or hallucinations. His physical examination is notable for a slightly elevated pulse rate and blood pressure as well as markedly dilated pupils bilaterally. The result of his urine toxicology screen is positive for cocaine. A. Bipolar disorder, manic B. Major depression C. Mood disorder due to a general medical condition D. Substance-induced mood disorder E. Adjustment disorder with depressed mood

D. The most likely diagnosis for this man is substance (cocaine)-induced mood disorder. Although he presents with classic manic symptoms (irritable mood, decreased sleep, etc), he has no psychiatric or family history of mood disorder. His physical examination reveals several findings not necessarily consistent with mania, namely, elevated vital signs (pulse rate and blood pressure) and dilated pupils. The important factor in this case is his obvious cocaine use, which can produce symptoms mimicking those of acute mania.

24.1 A 42-year-old woman describes a 20-year history of numerous physical complaints, including joint pain, dysuria, headaches, chest pain, nausea, vomiting, irregular menses, and double vision. Although they do not all occur at the same time, she has been suffering from one or more of these problems throughout her adult life. Many workups have been done for her, and she has undergone repeated hospitalizations, but no specific cause has yet been found. She is extremely anxious and has become significantly disabled as a result. Which of the following is the most likely diagnosis? A. Body dysmorphic disorder B. Hypochondriasis C. Pain disorder D. Somatization disorder E. Conversion disorder

D. The most likely diagnosis for this woman is somatization disorder. She presents with numerous somatic complaints, related to several body areas, which are not fully explained by a medical cause. The focus is on the symptoms themselves, not on a perceived physical defect (as in body dysmorphic disorder), on the fear of having a specific disease (as in hypochondriasis), or on symptoms of pain (as in pain disorder).

40.3 A 43-year-old divorced female veteran with schizophrenia is being followed in an outpatient community mental health clinic after being discharged from the hospital. Her medications have subsequently been increased to risperidone 3 mg in the morning and 4 mg in the evening. She has some paranoia and ideas of reference, but she denies auditory or visual hallucinations. Her mental status examination is significant for moderate psychomotor slowing, with little spontaneous speech, but with a coarse tremor of her hands bilaterally. Her stated mood is "fine," although her affect appears blunted, with little expression. Her gait is wide based and shuffling. A. Acute dystonic reaction B. Akathisia C. Neuroleptic malignant syndrome D. Parkinsonism E. Tardive dyskinesia

D. The patient is a middle-aged female veteran with chronic psychotic symptoms, just released from the hospital with an increase in her risperidone dose. She now demonstrates bradykinesia, shuffling gait, masked facies, and a coarse tremor, all consistent with antipsychotic-induced parkinsonism. Risk factors for the development of parkinsonism include female gender and an older age.

37.1 A 33-year-old married physician presents to your primary care practice with complaints of "depression." On interview, he denies pervasive feelings of sadness or anhedonia, and he has not had any change in appetite or weight, or any problems concentrating. He has felt tired much of the time for the past 6 weeks, with ongoing, multiple awakenings during the night. On further questioning, he reveals that these difficulties began when he was involved in a malpractice suit after the death of a patient. He was "up obsessing about it" prior to the trial when his sleep disturbance began. Although the suit was dropped, he continues to wake up frequently, worrying about not being able to fall back asleep. He denies medical problems, alcohol, or drug use. Which of the following is the most likely diagnosis for this patient? A. Breathing-related sleep disorder B. Circadian rhythm sleep disorder C. Major depressive disorder D. Primary insomnia E. Narcolepsy

D. The patient likely suffers from primary insomnia. There is no evidence of sleep apnea or a disconnect between the environment and the circadian rhythm of the patient. Despite the insomnia and fatigue, there is no pervasive depressed mood, anhedonia, or other neuro-vegetative symptoms suggestive of a major depression.

38.3 For a diagnosis of somatization disorder, which of the following criteria must be met? A. External motivation for symptoms (avoid work or financial gain). B. Patient is without significant impairment. C. Symptoms are intentionally feigned or produced. D. Symptoms begin prior to age 30. E. Symptoms last less than 6 months.

D. The symptoms must occur prior to age 30 and be present for several years.

32.1 A 63-year-old woman returns to her family physician with continuing headaches for 9 months. She describes the pain as "constant ... always with me," around her entire scalp. She does not appreciate much variation throughout the day, and she cannot name any aggravating or alleviating factors. Although she occasionally feels light-headed when in severe pain, she denies photophobia, visual changes, nausea, or vomiting. She is especially upset about the headaches as she retired in the past year and has been unable to visit her infant granddaughter. Complete neurologic examinations, computerized tomography, magnetic resonance imaging, laboratory studies, and lumbar punctures have been unremarkable. Which of the following is her most likely diagnosis? A. Factitious disorder B. Hypochondriasis C. Malingering D. Pain disorder E. Somatization disorder

D. This patient presents with criteria for pain disorder. She has chronic, unremitting headaches that are the focus of her complaints. They have interfered with her ability to travel, and the onset seems to coincide with her retirement and new grandchild. Her condition is not intentionally produced as in factitious disorder or malingering, nor is there any appreciable secondary gain (avoidance of work, financial compensation, etc). The concern is not on having a serious medical illness as in hypochondriasis or multiple physical complaints as in somatization.

52.1 Which of the following signs is most characteristic of early alcohol withdrawal? A. Decreased blood pressure B. Hypersomnia C. Persistent hallucinations D. Tremor E. Increased appetite

D. Tremor is the most characteristic sign of alcohol withdrawal. Vital signs are elevated in alcohol withdrawal because of autonomic hyperactivity. Patients generally have insomnia as a result, not hypersomnia. Hallucinations associated with alcohol withdrawal usually resolve within a week, while those occurring in delirium tremens usually resolve with the delirium. Cocaine withdrawal more typically involves hypersomnia and hyperphagia.

52.2 In what time frame after cessation of all drinking is he at most risk for delirium tremens? A. 6 to 8 hours B. 8 to 12 hours C. 12 to 24 hours D. 24 to 72 hours E. Over 1 week

D. Withdrawal symptoms usually, but not always, occur in stages: tremulousness or jitteriness (6-8 hours), psychosis and perceptual symptoms (8-12 hours), seizures (12-24 hours), and DTs (24-72 hours, up to 1 week).

25.1 A 39-year-old man is evaluated by mental health services in prison. He has a history of multiple arrests as both an adult and as a juvenile. After several interviews, a diagnosis of antisocial personality disorder is confirmed. He has a history of multiple psychiatric hospitalizations after suicide attempts and was in special education programming as a child. Which psychiatric diagnosis is most likely to have occurred comorbidly in such an individual? A. Attention-deficit/hyperactivity disorder B. Cocaine dependence C. Traumatic brain injury D. Major depression E. Conduct disorder

E. Although all the diagnoses listed are frequently comorbid, evidence of a diagnosis of conduct disorder with onset before age 15 is required for a diagnosis of antisocial personality disorder.

51.3 The patient above in Question 51.2 is begun in psychodynamic psychotherapy. Development of which of the following would be a reason to discontinue the therapy? A. Anger directed at the therapist B. Erotic feelings toward the therapist C. Missing sessions D. Regression E. Regular use of drugs or alcohol

E. Although it is not unusual for individuals with anxiety or depression to use alcohol or illicit drugs, regular use would interfere with the cognitive processing and attention required in profound therapy. In addition, the relevant feelings and conflicts addressed in psychotherapy would be enhanced, "masked," or distorted by the use of psychoactive substances. Emotions such as anger, love, and even sexual attraction are expected in insight-oriented therapies such as psychodynamic psychotherapy and psychoanalysis; as with all thoughts and feelings, they are examined by the therapist and patient during therapy. Because of the powerful emotions generated and the intense relationship between therapist and patient, it is typical for individuals to regress during the session (symptoms to temporarily worsen), or to "act out" (express an impulse or fantasy indirectly through behavior). This can manifest by missing appointments, arriving late, or forgetting to pay the bill.

11.1 A diagnosis of GAD is made for a 41-year-old female accountant who has had moderate but distressing symptoms for several years. She has strong negative feelings about taking any psychotropic medication, even after being educated about pharmacologic treatment options. Which of the following would be the most efficacious treatment option? A. Cognitive-behavioral therapy only B. Psychoanalysis only C. Psychoeducation only D. Supportive psychotherapy only E. Cognitive-behavioral therapy with an SSRI, venlafaxine, or buspirone

E. Because the patient's symptoms are moderate and she feels so negatively about medication, the best option for psychotherapy is CBT, the most-studied evidence-based psychotherapy. Other interventions such as insight-oriented psychotherapy and supportive psychotherapy are less likely to be effective. This should be combined with a non-addictive medication for anxiety.

48.4 In controlled studies, which of the following medications are ineffective for use in childhood anxiety disorders? A. Venlafaxine B. SSRIs C. Buspirone D. Tricyclic antidepressants E. Benzodiazepines

E. Benzodiazepines have not shown efficacy in controlled trials in childhood anxiety disorders.

48.2 Children or adolescents with separation anxiety disorder are at higher risk for which other psychiatric disorder? A. Malingering B. Somatization disorder C. Bipolar disorder D. Learning disability E. Major depression

E. Children and adolescents with separation anxiety disorder often present with or later develop symptoms of major depression. In children, this can include a depressed, sad, or irritable mood over an extended period of time.

31.1 A 25-year-old man comes to the hospital with symptoms of gastrointestinal distress, muscle aches, rhinorrhea, lacrimation, and an anxious mood. He states that he "wants to kick this thing once and for all." Which of the following medications would be most helpful in ameliorating his symptoms? A. Antabuse B. Haloperidol C. Naloxone D. Lorazepam E. Clonidine

E. Clonidine can be used to help ease the withdrawal symptoms of opioid withdrawal. It is not an opioid and does not have any addictive properties. However, the withdrawal may not be as painless as it would be if methadone were used. Blood pressure levels should be monitored when clonidine is used. Antabuse or disulfiram is a treatment option for alcoholics. It is not used to treat acute opioid withdrawal. Naloxone is used to counter the effects of life-threatening depression of the central nervous system and respiratory system from opioid overdose (eg, heroin or methadone overdose). It would worsen this patient's withdrawal symptoms. Lorazepam, a benzodiazepine, is commonly used in the treatment of alcohol withdrawal. Haloperidol, an antipsychotic, has no use in treating withdrawal.

60.4 A 22-year-old man with BPD loses his job at a local restaurant, the first job he held for longer than a month. His mother dies suddenly 3 weeks later. One month after his mother's death, the patient tells his therapist, whom he has been seeing once a week, that he has trouble sleeping and wakes up at 3 AM and is unable to go back to sleep. He has lost 13 lb in 5 weeks without trying to do so. He reports low energy and a decreased interest in his usual hobbies. He states that he feels depressed but then grins and says, "But I'm always depressed, aren't I?" Based on his history, which of the following should the clinician do next? A. Ask the patient to keep a sleep log. B. Begin seeing the patient for daily psychotherapy. C. Hospitalize the patient. D. Start treating the patient with a mood stabilizer such as carbamazepine. E. Start treating the patient with an antidepressant such as paroxetine.

E. Comorbidity of major depression with BPD is quite common. When vegetative symptoms or other qualitative changes occur, medication for the major depression is necessary. This is often a difficult call to make, since borderline personality disorder patients present as "stably unstable," that is, their lives are often disruptive and chaotic and they often have severely depressed moods. Changes in sleep or appetite are often the best clues as to when a BPD patient has developed a concurrent major depression.

32.2 Which of the following would be the most useful approach for the patient in Question 32.1? A. Confrontation regarding the psychological nature of her pain. B. Prescribe non-narcotic pain medication. C. Reassurance that there is no evidence of pain. D. Referral to a mental health professional. E. Validation of her experience of pain.

E. One of the most important aspects in management of this disorder is to validate the patient's experience of pain. An empathic response will serve to strengthen the therapeutic alliance. Conversely, implying that the symptoms are "not real" or denying that "there is anything wrong" will only cause the patient further distress and can actually worsen the pain. Although referral to a mental health professional can be indicated and helpful given the psychological factors present in pain disorder, this subject should first be gently broached with the patient in order to avoid the appearance of not taking the pain seriously. Analgesic medication is generally not helpful and narcotic analgesics should be avoided given their abuse and withdrawal potential.

10.3 Which of the following would be the most useful psychiatric treatment for the patient in Question 10.2? A. Antianxiety medication B. Antidepressant medication C. Antipsychotic medication D. Electroconvulsive therapy E. Group psychotherapy

E. Psychotherapies, such as insight-oriented, family, behavioral, and group therapy have been useful in patients with dependent personality disorder. Medications for depression, anxiety, and/or psychosis, or electroconvulsive therapy (ECT), would only be indicated if the patient had a comorbid psychiatric illness, not evident in the patient in Question 10.2.

22.2 Patient is evaluated fully and determined to have dysthymic disorder. Which of the following medications is the most appropriate first-line treatment for her? A. Desipramine B. Lithium C. Lorazepam D. Phenelzine E. Sertraline

E. SSRIs (such as sertraline), SNRIs, and bupropion have demonstrated efficacy in treating dysthymia. Although TCAs and MAOIs are also beneficial, newer antidepressants such as SSRIs or SNRIs are better tolerated and safer in overdose. Neither lithium nor lorazepam is indicated for dysthymic disorder

3.3 A 25-year-old woman describes a lifelong history of being "scared of heights." She becomes uncomfortable when at an elevation higher than three stories and whenever traveling or shopping becomes preoccupied with knowing the heights of buildings. On finding herself at a significant distance from the ground, she has severe anxiety symptoms including trembling, lightheadedness, numbness and tingling, and a fear of dying. Which of the following is the most likely diagnosis? A. Generalized anxiety disorder B. Panic disorder with agoraphobia C. Panic disorder without agoraphobia D. Social phobia E. Specific phobia

E. Specific phobia is the most likely diagnosis for this woman. Although she has panic attacks, they are not unexpected and result from being in an elevated location. Her fears are of a particular situation (heights) rather than of having further panic attacks.

46.3 In the patient from Question 46.2, his play with tops and balls might be considered a stereotyped behavior. Which of the following statements best defines stereotypical behavior? A. Interacting preferentially with other children of a specific race B. Preference for similar schedules of play every day C. Focus of play on only one aspect of a toy D. Difficulty playing with a toy creatively beyond its most obvious use E. Spinning or repetitively using a toy in a specific manner for extended periods of time

E. Spinning or repetitively using a toy in a specific manner for extended periods of time is characteristic of stereotyped play. Autistic individuals often play with toys or objects in odd or eccentric ways. A typical example of stereotyped behavior in playing with toys is spinning or moving an object or toy in the same way for long periods of time.

43.4 A 29-year-old patient complains of sudden episodes in which she experiences tachycardia, an intense sense of fear, paresthesias, tremors, and shortness of breath that begin suddenly and cease after 5 to 10 minutes without any other intervention. She has three or four per week. The most likely diagnosis is which of the following? A. Myocardial infarction B. Asthma C. Hyperthyroidism D. Agoraphobia E. Panic attacks

E. Tachycardia, derealization, paresthesias, and shortness of breath are the classic presentation for panic attacks. Panic attacks are also associated with agoraphobia but are distinct from that disorder.

30.3 Acute stress disorder is diagnosed in a 32-year-old woman who witnessed her fiancé being shot to death in a robbery attempt. She has difficulty sleeping and feels that she is not emotionally attached to anything around her. She also has repetitive flashbacks of the event and avoids going near the location where the incident occurred. Which of the following medications might be helpful to this patient over the short term? A. Buspirone B. Paroxetine C. Risperidone D. Valproate E. Zolpidem

E. The use of a hypnotic for insomnia is likely to be helpful to this patient in the short term. Buspirone is used for those patients diagnosed with generalized anxiety disorder, while paroxetine is used in the treatment of major depression. Risperidone is an antipsychotic and valproate is used for mood stabilization of patients with bipolar disorder.

15.1 Which of the following is an associated disorder in the family histories of patients with schizotypal personality disorder? A. Alcohol dependence B. Bipolar disorder C. Hypochondriasis D. Panic disorder E. Schizophrenia

E. There is a greater association of cases of schizotypal personality disorder among biological relatives of patients with schizophrenia than among controls. The other disorders listed are not associated with higher rates of personality disorders.

29.4 Which of the following treatment options would be contraindicated among recommended treatment options for the patient in Questions 29.2 and 29.3? A. Nutritional rehabilitation B. Cognitive-behavioral psychotherapy C. Careful use of SSRIs D. Group psychotherapy E. Atypical antipsychotic medications to increase her appetite

E. There is no clinical evidence for the use of atypical antipsychotics in a patient with bulimia.

36.2 A parent brings his child to the pediatrician's office because of concerns regarding sleep. The child is 3 years old and often wakes up at night screaming loudly, appearing very frightened, striking out when touched, and inconsolable. She remembers none of this when she wakes the next morning. Which of the following pathologies is likely associated with this condition? A. Posttraumatic stress disorder B. Major depressive disorder C. Acute asthma D. Acute cystitis E. Restless leg syndrome

E. This case is typical for sleep terrors. Sleep terrors often occur in the presence of another sleep disorder—in particular, restless leg syndrome and sleep-disordered breathing.

40.1 A 50-year-old woman with a diagnosis of schizoaffective disorder, bipolar type, complains of "nervous tics." She denies significant affective symptoms but complains of chronic auditory hallucinations of "whispers" without commands. No suicidal or homicidal ideation is present. On examination, she is noted to be sticking her tongue in and out of her mouth and to have repetitive, rhythmic movements of her hands and feet. A. Acute dystonic reaction B. Akathisia C. Neuroleptic malignant syndrome D. Parkinsonism E. Tardive dyskinesia

E. This female patient with chronic schizoaffective disorder now demonstrates choreoathetoid movements of her tongue and extremities, consistent with tardive dyskinesia. She has several risk factors for development of tardive dyskinesia, including likely long-term treatment with antipsychotics, female gender, and a mood disorder.

47.4 A 35-year-old woman is engaged in psychotherapy to address her avoidant personality disorder. In particular, she is distressed by her inability to maintain a romantic relationship with a man. During the course of treatment, the therapist learns that her father was alcoholic, and was physically abusive to the patient and her mother. Which defense mechanism best describes the patient's behavior? A. Undoing B. Splitting C. Isolation of affect D. Idealization E. Displacement

E. This patient can be theorized to be using displacement to assume that all men will act as punitively toward her as her father did. Displacement and projection are the two defense mechanisms most commonly utilized by patients with avoidant personality disorders.

31.3 A 32-year-old man with a long-standing heroin addiction has recently started maintenance treatment with methadone. Three days after starting the methadone regimen, he is experiencing some craving, diarrhea, and mild sweating. His urine toxicology screen is negative for any opiates besides methadone. Which of the following is the most appropriate course of action? A. Increase the dose of methadone. B. Decrease the dose of methadone. C. Keep the dose of methadone the same and assure the patient that the symptoms will subside. D. Write a prescription for clonidine to be taken along with the methadone. E. Put the patient on a 1-week methadone taper program and refer him to Narcotics Anonymous.

A. Clinical signs of withdrawal appearing very early in the methadone maintenance treatment of heroin addiction are an indication that the dose is not sufficient to ameliorate all the withdrawal symptoms. Because the patient is at great risk of returning to the use of heroin at this point in the process, a dose adjustment upward in order to prevent craving and withdrawal would be appropriate.

50.3 A 23-year-old man is referred from an outpatient drug rehabilitation program to a psychiatrist for depression. The patient endorses having used "everything I can get my hands on, as often as I can get it." A drug screen is positive for a substance. The psychiatrist, however, doubts that the drug identified is responsible for the patient's symptoms of depression. Which of the following drugs is most likely to be present on the drug screen? A. Cannabis B. Cocaine C. Alcohol D. Methamphetamine E. Inhaling spray paint or "huffing"

A. DSM-IV does not recognize cannabis-induced mood disorders. Mood disorders can occur with almost any other abused substance, as can psychotic disorders and anxiety disorders, though clearly some are more common than others, given the substance.

60.1 A 24-year-old woman with BPD is admitted to a psychiatric hospital because of suicidal ideation. The physician on call tells the patient about all the rules and regulations in the unit and that "although it is a great place to get better, it is a lot of work." Which of the following is this physician attempting to do with this patient? A. Decrease idealization of the unit and the hospitalization. B. Discourage the patient from splitting. C. Dissuade the patient from signing in voluntarily. D. Encourage the patient to seek admission elsewhere. E. Investigate the patient's motivation for desiring admission.

A. Decreasing idealization of the unit before admission will help decrease the devaluation that inevitably follows. Since these swings (between idealization and devaluation) are so typical in borderline personality disordered patients, it is useful to minimize them right from the time of admission. Otherwise, they can become quite disruptive on an inpatient unit, as the patient first comes to the unit believing he/she can be completely and perfectly taken care of, followed by intense anger when devaluation follows and the patient believes the inpatient unit is incompetent and useless.

60.3 A 24-year-old woman is seen in the emergency department after superficially cutting both her wrists. Her explanation is that she was upset because her boyfriend of 3 weeks just broke up with her. When asked about other relationships, she says that she has had numerous sexual partners, both male and female, but none of them lasted more than several weeks. Which type of psychotherapy might she be most likely to respond to? A. Dialectical behavioral therapy B. Interpersonal psychotherapy C. Parent assertiveness training D. Psychopharmacotherapy E. Supportive psychotherapy

A. Dialectical behavioral therapy is a form of cognitive therapy shown through controlled studies to be effective in treating BPD. This therapy attempts to help patients explore their own behavior, thoughts, and feelings in the present without delving into the patient's childhood, which tends to be regressive for these patients, resulting in increased suicidal behavior and acting out.

34.3 A 27-year-old woman and her 7-year-old son present to a mental health center for treatment. The patients were passengers in the back of the family car, when they were struck by a semitractor trailer, which killed the father and an older sister. Both mother and son endorse significant depressive symptoms. Which of the following symptoms would most likely differ between the presentations of these two patients? A. Irritability B. Suicidal thoughts C. Flashbacks D. Insomnia E. Inattention

A. In the clinical presentation of children and adolescents, one will often find evidence of irritability or short temper rather than a feeling of sadness or depression. The ability to understand the concept of depression seems to be developmentally mediated.

42.2 A 9-year-old child presents with a history of motor and vocal tics with obsessive-compulsive symptoms that is worse in the winter and early spring months. Which of the following pathologies would be most important to rule out before starting treatment of Tourette disorder? A. Streptococcal infection B. Environmental allergies C. Autism D. Marijuana abuse E. Rett disorder

A. Investigating for a possible case of PANDAS is important in this child and rapid treatment of streptococcal infections and prevention of reinfection may impact the clinical course of such children.

42.1 When examining a patient with possible Tourette disorder, you should inquire carefully about a family history of which of the following disorders? A. Obsessive-compulsive disorder B. Sleep terror disorder C. Primary insomnia D. Developmental disability E. Parkinson disease

A. Obsessive-compulsive disorder is far more common in families of patients with Tourette disorder than in the general population. 50% of patients with Tourette have significant obsessive-compulsive symptoms.

47.2 A 24-year-old man presents to a therapist. Which of the following statements made by the patient is most consistent with avoidant personality disorder? A. "I have a couple of close friends, but it seems like I just don't get most people." B. "I'm usually fine around people. It's just when I'm around a lot of people I've never met before that I get freaked out." C. "I'm afraid that people are plotting against me." D. "My mom thinks I have a problem with people. I can take them or leave them." E. "My girlfriend thinks I have a problem with people, like with her friends. What do you think?"

A. Option A is most consistent with avoidant personality disorder. The patient has some close relationships and seems to desire more, but does not feel that he can maintain such relationships. B is more consistent with social phobia. C is grossly paranoid, consistent with a psychotic disorder. D is highly suggestive of schizoid personality disorder, as the patient has a close relationship with his parent but has no interest in other human contact. E suggests dependent personality disorder.

41.2 The patient in Question 41.1 would be most likely to become depressed after which of the following life occurrences? A. Aging B. Graduation C. Job change D. Marriage E. Moving to a new city

A. Patients with narcissistic personality disorder do not handle aging well because beauty, strength, and youth are often highly valued. Any blow to their fragile (but covert) self-esteem can raise their feelings of envy and anger, and subsequently lead to depression. All the other life occurrences represent changes which may be stressful to people, but do not affect those with personality disorders any differently than the general population (other than the fact that patients with personality disorders do not handle any kind of stressors well in general).

41.3 A 36-year-old man with narcissistic personality disorder calls your office asking for an appointment with the "best therapist in the clinic." One of his complaints is difficulties in his relationships with his colleagues. The patient states "They are not giving him the credit he deserves for his accomplishments at the law firm." What is the most likely reason the patient is seeking treatment? A. Anger B. Anxiety C. Attempting to identify with others D. Grandiose thinking E. Seeking medication

A. Patients with narcissistic personality disorder rarely seek treatment and tend to have little insight into their grandiosity. When these individuals do present for treatment it is usually due to underlying anger or depression resulting from being belittled or not receiving the admiration to which they feel entitled.

8.3 Social phobia differs from specific phobia in which of the following ways? A. Focus or nature of the fear B. Duration of the illness C. Absence of panic attacks D. Degree of avoidance of the situation E. Recurrent nature of the fear

A. Social phobia might be also considered a specific phobia under some circumstances and is very similar in duration, progression, symptoms, and patient avoidance. The real difference is the nature of the fear—in social phobia this is characterized by social situations.

34.2 A 52-year-old man presents to his primary care physician after the death of his wife from breast cancer 2 months ago. He complains of depression, inconsolable sadness, frequent crying, and an inability to focus upon his work and usual activities. Which of the following treatments would likely be most helpful for him? A. Supportive psychotherapy B. Family therapy C. A selective serotonin reuptake inhibitor antidepressant D. Psychoanalysis E. Behavioral modification therapy

A. Supportive psychotherapy is indicated to help the patient deal with his response to his loss, either in an individual or a group setting. Medications are not indicated for bereavement, except perhaps a mild sleep aid if insomnia is a problem. Behavioral modification and/or psychoanalysis are both unnecessary in this setting, since a much more acute problem which is not behavioral in nature is at issue. The patient does not indicate familial problems (other than his wife's death) so supportive therapy is the best option.

45.3 A 38-year-old woman comes in for evaluation of an abscess on her thigh. Her chart documents frequent outpatient and hospital visits. She is admitted, her abscess is drained, and she is treated with antibiotics. Culture studies demonstrate microorganisms consistent with fecal matter, and a further physical examination reveals many old scars, presumably self-inflicted. A. Factitious disorder B. Malingering C. Conversion disorder D. Somatization disorder E. Hypochondriasis

A. The most likely diagnosis for this woman is factitious disorder. She presents with a self-induced infection, as well as a history of frequent utilization of hospitalizations and other medical services. Her illnesses are consciously created, without a desire to obtain obvious external gain other than assumption of the patient role.

30.1 A 28-year-old male assembly-line worker comes in for treatment after developing symptoms a few days after a serious factory accident in which he was knocked unconscious by a machine. He was medically cleared directly following the event but later developed nightmares about the accident. He says he has been very anxious, fears returning to work, and is thinking about seeking disability pay. He has refused to talk to his wife about the incident, because it "makes it worse." Being around large objects also frightens him, since he is worried he will be hit in the head again and he notes that he thinks about the accident constantly. What is the most likely diagnosis? A. Acute stress disorder B. Adjustment disorder C. Factitious disorder D. Generalized anxiety disorder E. Posttraumatic stress disorder

A. The most likely diagnosis is ASD, since this patient's symptoms occurred within 4 weeks of the event and have lasted at least 2 days. If the symptoms had lasted longer than 4 weeks, the diagnosis for a patient with these symptoms would become PTSD. An adjustment disorder is characterized by mood disturbances (anxiety, depression) in response to a difficult situation, but adjustment disorders do not present with the avoidance symptoms (avoiding talking about the incident) and/or reexperiencing of the traumatic event. Generalized anxiety disorder presents with the patient constantly worrying about a wide variety of imagined problems, not connected to an individual traumatic event. Factitious disorder is the feigning of mental or physical illness for primary gain.

43.1 A 62-year-old man with a history of diabetes mellitus, chronic COPD, hepatitis C, peripheral neuropathy, and a pacemaker for control of cardiac arrhythmia complains of new-onset episodic anxiety occurring over the past 3 weeks. He has no history of anxiety symptoms. Episodes of intense anxiety tend to occur in the daytime, last for 30 minutes to 1 hour, and are accompanied by hyperventilation and a sense of "palpitations," as well as some confusion and disorientation. Which of the following should be excluded from your differential diagnosis? A. Panic disorder B. Episodic hypoglycemia C. Hypoxia caused by COPD D. Hypoxia caused by arrhythmia E. Hepatic encephalopathy

A. This man has multiple medical problems that can cause anxiety symptoms and no history of anxiety. He can be inadvertently overdosing on insulin or hypoglycemic medications, his pacemaker may not be functioning properly, he may have transient hypoxia, or he may have hepatic encephalopathy. Although a primary diagnosis of an anxiety disorder is possible, it is the least likely in this case.

43.2 A 45-year-old man with schizophrenia, type II diabetes, and alcohol and cocaine dependence comes to the emergency department 2 hours after drinking half of a fifth of whiskey and smoking cocaine, after which he fell and hit his head. He describes losing consciousness for several minutes. He states that he feels extremely anxious, saying, "I can't calm down." In the emergency department, he is noted to be hyperventilating, diaphoretic, and complains of nausea. Which of the following tests should be performed immediately? A. Blood glucose level determination B. Thyrotropin (TSH) level determination C. Abdominal ultrasound examination D. Test for human immunodeficiency virus (HIV) disease E. Urine toxicology screen

A. This patient is hypoglycemic, or his anxiety is caused by alcohol withdrawal or cocaine intoxication. Because he has history of a head injury, a computed tomography scan of the head should be performed to rule out a hemorrhage. The other laboratory or imaging tests are unnecessary in the short term. Though not likely to be associated with the shortness of breath, getting B12 and folate levels soon after admission would also be helpful because of his history of alcohol use.

39.3 A 24-year-old woman gives birth to her first child, a boy. Four days after the delivery, her husband calls his wife's obstetrician to tell him that he is concerned about his wife's behavior. The husband states his wife sits by herself in the bedroom almost continually, appearing dazed, but also easily irritated and crying frequently. She tends to the baby only if reminded to do so by her husband, and when she holds the baby, she does so stiffly. She is awake most of the night, even when the baby is sleeping. The husband claims she has told him that their baby is "evil" and that she is "satanic" for having the child. Which of the following is her most likely underlying diagnosis? A. Bipolar disorder B. Borderline personality disorder C. Major depressive disorder D. Schizoaffective disorder E. Schizophrenia

A. This woman is most likely suffering from postpartum psychosis. Because of the risk to the mother and infant, postpartum psychosis is a psychiatric emergency, requiring immediate hospitalization. Most cases of postpartum psychosis are due to bipolar disorder. Major depressive disorder and primary psychotic illnesses such as schizophrenia or schizoaffective disorder are much less likely the cause of postpartum psychosis.

1.1 A 70-year-old woman presents to her primary care provider complaining of fatigue for the past 7 weeks. She admits to difficulty falling asleep, a poor appetite with a 10-lb weight loss, and thoughts of wanting to die. She admits to having had symptoms similar to these on several occasions in the past, but "never this bad." Her medical problems include asthma and high cholesterol. She uses an albuterol inhaler only as needed. Which of the following symptoms is necessary in order to make a diagnosis of major depressive disorder? A. Depressed mood B. Decreased appetite C. Excessive guilt D. Fatigue E. Suicidal ideation

A. Although a change in appetite, decreased energy, fatigue, and suicidal ideation are all criteria used in diagnosing major depressive disorder, one of the symptoms must be either a depressed mood or anhedonia.

2.1 A 54-year-old, psychotic, homeless man is brought into the emergency department by the police for medical clearance, after being picked up for aggressive behavior and disorderly conduct. He has a history of alcoholism and substance abuse, as well as poorly controlled diabetes and hypertension. Which of the following symptoms is most specific to a diagnosis of schizophrenia, as opposed to other etiologies of psychosis? A. Auditory hallucinations B. Belief that one has the power of an alien species C. Catatonic symptoms D. Depression E. Inappropriate affect

B. Although all these symptoms can be seen in various psychotic disorders, the presence of a bizarre delusion is the most specific to schizophrenia. Only one psychotic symptom is needed to diagnose schizophrenia if there are bizarre delusions, auditory hallucinations commenting on the patient, or two or more voices speaking to each other.

12.3 A 24-year-old woman with a diagnosis of bipolar disorder, manic, is treated with a mood stabilizer (lithium) and haloperidol, an antipsychotic. Which of these medications should be discontinued first once her condition has been stabilized? A. Both should be discontinued simultaneously once the patient's condition is stable. B. Haloperidol because of the risk of extra pyramidal side effects. C. Haloperidol because of the risk of habituation. D. Lithium because of the risk of renal damage. E. Lithium because of the risk of weight gain.

B. Because tardive dyskinesia is irreversible, and the risk of its appearance increases with continued use, antipsychotics should be discontinued as soon as the patient's psychotic symptoms remit, and not used again as long as the patient's condition remains stable.

11.2 A 38-year-old man without past medical history presents to his family physician with the chief complaint of "I'm having ulcers." His history, however, does not appear to be consistent with gastritis, ulcers, or reflux. After further questioning, he describes ongoing headaches for 8 months, along with difficulty sleeping. Although he denies any specific or recent stressors, he admits to "always being a worry-wart," worrying about many different aspects of his life. When asked to describe his mood on most days, he replies, "I've been depressed my whole life." He only has been taking ranitidine over the counter as needed. He drinks one to two glasses of wine 1 to 2 days per week and denies drug use. Based on his likely diagnosis, which of the following additional psychiatric disorders are most likely to also be present in this patient? A. Antisocial personality disorder B. Dysthymic disorder C. Factitious disorder D. Schizophrenia E. Somatization disorder

B. This patient appears to suffer from generalized anxiety disorder. There is a high comorbidity in this illness, especially with other anxiety disorders, such as panic disorder and phobias. Dysthymic disorder and major depressive disorder are also very commonly associated conditions. Substance use disorders are also not uncommon in patients with GAD.

17.2 A 52-year-old man is hospitalized for triple CABG and subsequently develops a delirium. He has a history of one closed head injury with loss of consciousness, as well as past alcohol abuse with 7 years of sobriety under his belt. Which of the following factors is most likely the prime contributory factor in the development of his delirium? A. His age B. Hospitalization C. Status post-cardiac surgery D. History of head injury E. History of alcohol abuse

C. Advanced age is a major risk factor, with 60% of nursing home residents over age 75 experiencing repeated episodes of delirium. Of all medically ill, hospitalized patients, 10% to 30% exhibit delirium. However, some studies indicate that 90% of postcardiotomy patients experience delirium. Preexisting brain damage, prior history of delirium, alcohol dependence, diabetes, cancer, sensory impairment, and malnutrition may also contribute.

20.2 A 23-year-old medical student makes lists of all the tasks that he must accomplish each day. He spends hours studying and refuses to go out with his colleagues even when there are no tests on the immediate horizon, preferring to spend his time looking at specimens in the laboratory. He keeps meticulous notes during all his classes and prefers to attend every lecture, not trusting his colleagues to take notes for him. He is doing well in school and has a girlfriend who is also a medical student. Which of the following disorders does this student most likely have? A. Obsessive-compulsive disorder (OCD) B. Obsessive-compulsive personality disorder C. Obsessive-compulsive traits D. Schizoid personality disorder E. Paranoid personality disorder

C. Although this student clearly demonstrates some traits of obsessive-compulsive behavior, his social and occupational functioning both are good, which rules out the personality disorder.

19.2 Over the next 2 years, this patient continues to struggle and is arrested for truancy, fighting, and possession of drug paraphernalia. Given this information, what might we best be able to say about patients with this diagnosis in general? A. Patients are more likely to have parents with a history of schizophrenia. B. Patients are more likely to be female. C. Patients are more likely to have parents who have antisocial personality disorder and alcohol dependence. D. The presence of this pathology is independent of social class. E. The course of this disease is time limited and she is likely to grow out of it.

C. This patient now fits the criteria for conduct disorder. Conduct disorder is more common in children of parents with antisocial personality disorder and alcohol dependence than it is in the general population.

17.1 A 71-year-old woman with a history of early Alzheimer disease is hospitalized with pneumonia. During the course of the hospitalization, her family and primary physician notice a distinct worsening of her memory and alertness. Which of the following procedures would be the most sensitive in establishing a diagnosis of delirium? A. Chest radiograph B. Computerized tomography scan of the brain C. Electrocardiogram D. Electroencephalogram (EEG) E. CBC with differential

D. Although the other studies are all helpful in determining the etiology of delirium, only an EEG is sensitive in diagnosing this disorder. In almost all cases of delirium, an EEG shows generalized slowing. In cases where alcohol or sedative-hypnotic withdrawal is causing delirium, an EEG can show fast low-voltage activity. In hepatic encephalopathy, an EEG characteristically displays triphasic delta waves. Electroencephalogram findings typically remain normal early in the course of Alzheimer disease.

26.3 A 40-year-old man with schizoaffective disorder has been hospitalized in an inpatient psychiatry unit for the third time in the last 5 years. During each episode, he becomes noncompliant in taking his medications, develops manic symptoms and auditory hallucinations, and then becomes violent. In the inpatient unit, he physically threatens other patients and staff and is generally agitated. He is put in isolation to help quiet him. The patient is prescribed a mood stabilizer. Which of the following medications might also help relieve this patient's acute agitation? A. Buspirone B. Fluoxetine C. Chloral hydrate D. Risperidone E. Benztropine

D. Atypical neuroleptics such as risperidone or quetiapine are shown to be effective in managing manic symptoms especially in the acute states while mood stabilizers reach therapeutic levels.

23.3 An MRI of the brain is performed for the patient in Question 23.1. It demonstrates numerous small infarcts throughout the deep white matter structures. Which of the following best describes the likely progression of his illness? A. No change B. Gradual improvement C. Gradual worsening D. Stepwise deterioration E. Rapid progression

D. Given his history of hypertension and lacunar white-matter infarcts, his diagnosis is most likely vascular (multi-infarct) dementia. Vascular dementia characteristically demonstrates a stepwise deterioration in cognitive functioning corresponding to either small ischemic events or emboli causing strokes. Alzheimer disease progresses gradually over many years, whereas dementias caused by head trauma or anoxia are generally sudden in onset and have a subsequently stable course. Dementias with certain infectious etiologies, such as Creutzfeldt-Jakob disease, have a fairly rapid rate of progression.

1.4 The woman in Questions 1.2 and 1.3 is seen 1 year later for a return visit. She has remained on the citalopram at the same dose, and she is tolerating it well, but she worries about "always having to take medication." What is her risk of recurrence if she is not maintained on antidepressant medication? A. 0% to 10% B. 10% to 25% C. 25% to 50% D. 50% to 85% E. 85% to 100%

D. The recommendations for maintenance therapy in major depressive disorder should be made on a case-by-case basis. However, the illness tends to run a chronic course, especially if treatment is discontinued. Indeed, 50% to 85% of individuals will suffer from at least one further episode, most likely within 2 to 3 years.

17.4 In the previous case, the patient is determined to have a delirium due to infection, overlying his dementia. Which of the following is the most important treatment approach in treating his delirium? A. Detection and correction of the underlying abnormality B. Environmental strategies to help with orientation C. Treatment with an antipsychotic medication for hallucinations D. Physical restraint to protect the patient from injury E. Treatment with a benzodiazepine to reduce agitation

A. Although environmental strategies and pharmacologic and physical interventions can be helpful and necessary to help orient patients or protect them from harm, the most essential treatment approach in all cases of delirium is to detect and correct the underlying cause of the disorder. The occurrence of an episode of delirium itself suggests a poor prognosis, meaning these patients have a significantly elevated future incidence of mortality. In the elderly, antipsychotic agents should be used with caution given some increased risk of mortality, and benzodiazepines may cause disinhibition, oversedation, or paradoxical excitation.

14.2 A 24-year-old woman states that her alcohol consumption is two glasses of wine or margaritas three times during the week and five vodka drinks on Friday and Saturday evenings. This pattern has been going on for nearly a year. She denies any impairment at work as a result of her drinking. She says she is "social" and goes out frequently. Further discussion reveals episodes of blacking out during drinking that have occurred five times in the previous 8 months. She describes these events as "having no recollection of leaving one bar and heading to the next." Which of the following questions is the most specific in screening this patient for alcohol dependence? A. Have you ever attempted to cut down on your drinking? B. How frequently do you drink? C. How much do you drink per day? D. When did you start drinking regularly? E. Do you drink hard liquor or beer and wine only?

A. Although questions regarding the specific frequency, kind of alcohol, amount, and initial use of alcohol are important in establishing a history and pattern of abuse, only asking about attempts to cut back is specific for alcohol dependence, as this question addresses the inability to control alcohol use. The CAGE questionnaire has been validated in screening for alcohol dependence. It uses the mnemonic CAGE. A "yes" answer to two or more of the following questions is a sensitive indicator in diagnosing alcohol dependence. Have you ever felt you should Cut down on your drinking? Have you ever felt Annoyed by someone criticizing your drinking? Have you ever felt Guilty about your drinking? Have you ever had an Eye opener (a drink first thing in the morning)?

28.1 A 9-year-old boy is referred to a psychiatrist because of poor school performance. He has been tested for learning disabilities but none are present, with an IQ in the high normal range. The teacher reports that it is hard to hold his attention. In addition, he appears hyperactive and fidgety at school, which disrupts the class. However, he does not purposefully go out of his way to disobey the teacher. His parents have noticed no difficulties at home, but his soccer coach has noticed attention problems during practice, and his Sunday school teacher has trouble teaching him because of distractibility. Which of the following is the most likely diagnosis for this patient? A. ADHD, combined type B. ADHD, predominantly hyperactive type C. ADHD, predominantly inattentive type D. Oppositional defiant disorder E. No diagnosis, because the ADHD symptoms must be reported in the home

A. Attention-deficit disorder, combined type. The diagnostic criteria for ADHD require that the symptoms be present in more than one setting, usually at home and at school. However, this child seems to have evidence of symptoms observed at school, church, and soccer. The child had prominent distractibility and hyperactivity.

8.2 Which of the following is the treatment of choice for social phobia? A. Behavior therapy B. SSRIs C. Electroconvulsive therapy D. Psychoanalysis E. Divalproex sodium (Depakote)

A. Behavior therapy is the treatment of choice for social phobia. Benzodiazepines can be used to reduce the associated anxiety. A beta-blocker such as propranolol can also help reduce the autonomic hyperarousal that occurs in social settings. In addition, some SSRIs are also useful.

17.3 An 82-year-old man with a history of vascular dementia is brought to the hospital for increased agitation and urinary tract infection (UTI). Which of the following features most distinguishes effects of a delirium from dementia? A. Altered level of consciousness B. Behavioral disturbances C. Cognitive deficits D. Disorientation E. Presence of hallucinations

A. Both delirium and dementia can result in behavioral disturbances, cognitive deficits, and poor orientation. However, in all cases of delirium there is an alteration (reduction) in the level of consciousness, whereas in dementia (in the early stages) there is an alert, stable level of consciousness.

2.3 A 78-year-old man presents with 4 weeks of significant depression following the sudden, unforeseen death of his wife of 35 years. He reports difficulty sleeping, a 10-lb weight loss, frequent crying spells, and profound guilt over surviving her. For the last several days, he has been convinced that his body is literally decaying. He admits to seeing his wife's face during the day, as well as hearing her voice telling him to kill himself and join her. A. Major depression with psychotic features B. Schizoaffective disorder C. Schizophrenia D. Psychosis secondary to a general medical condition E. Substance-induced psychotic disorder

A. The most likely diagnosis for this man is major depression with psychotic features. Significant depression and neurovegetative symptoms are present, as well as delusions and auditory and visual hallucinations. Although he has mood symptoms and psychotic symptoms, his history is consistent with major depression because his mood symptoms preceded his psychotic symptoms.

29.2 A 34-year-old woman presents with a 10-year history of episodes in which she eats large quantities of food, such as eight hamburgers and three quarts of ice cream, at a single sitting. Because of her intense feelings of guilt, she then repeatedly induces vomiting. This cycle repeats itself several times a week. She is extremely ashamed of her behavior but says, "I can't stop doing it." On examination, which of the following physical findings is most likely to be seen? A. Dental caries B. Lanugo C. Muscle wasting D. Obesity E. Body weight at less than the 10th percentile of normal

A. The most likely diagnosis for this woman is bulimia nervosa. Physical findings can include dental caries, a round face caused by enlarged parotid glands, or calluses on the fingers resulting from recurrent self-induced vomiting. Lanugo and muscle wasting result from the severe weight loss characteristic of anorexia nervosa.

5.3 Parents of a 10-year-old boy note that their son does well with his family until he is not allowed to do something he wants to do. When this occurs, he will get irritable, impulsively aggressive, and agitated for several hours. Once he calms, or gets his way, he is happy and pleasant again. At school he has no trouble focusing but if he does not want to do something he becomes argumentative. Which of the following is the most likely diagnosis? A. Oppositional defiant disorder B. Bipolar disorder-mania C. Attention deficit-hyperactivity disorder combined type D. Major depression E. Generalized anxiety disorder

A. The pattern described best fits oppositional defiant disorder, since the patient can be seen to calm down and be pleasant after he gets his way. A child with bipolar disorder might be seen to have these kinds of outbursts, but the mood dysregulation continues, even after the child "gets his way." Children in the midst of a bipolar disorder never "calm down and be pleasant." Depressed children would also have a persistent mood shift. The symptoms of inattention and distractibility are not present so the patient does not have ADHD. There are no signs of anxiety.

26.2 A 28-year-old man is brought to a psychiatrist complaining that he has been hearing voices for the past several weeks. He says that he also heard these voices 3 years ago. He notes that his mood is "depressed" and rates it 3 on a scale of 1 to 10 (with 10 being the best he has ever felt). He does not recall if his mood was depressed the last time he had psychotic symptoms. Which of the following actions should the physician take next? A. Obtain more detailed information about the time course of the psychotic symptoms and the mood symptoms. B. Treat the patient with an antipsychotic agent. C. Treat the patient with an antidepressive medication. D. Request a urine toxicology screening. E. Refer the patient to supportive psychotherapy

A. The time course of the mood symptoms and psychotic symptoms determines the treatment of the patient because the diagnosis can be schizoaffective disorder versus major depression. Although the patient should undergo a urine toxicology screening, this should not be done until a complete history is obtained so that further targeting of laboratory testing can be accomplished.

9.3 An obtunded, young woman is discovered by the police sitting in the middle of the street, and is subsequently brought to the emergency department. She is unable to verbalize any history. Which of the following sets of findings is most indicative of PCP intoxication? A. Nystagmus, muscle rigidity, cannabinoids present on urine toxicology B. Dilated pupils, bradycardia, runny nose C. Pinpoint pupils, tachycardia, orthostatic hypotension D. Ocular nerve palsy, cardiac arrhythmias, pseudobulbar palsy E. Hallucinations, heart block, lower limb weakness

A. The triad of nystagmus, muscle rigidity, and numbness points strongly to PCP intoxication. Other symptoms that can occur include hypertension or tachycardia, ataxia, dysarthria, seizures or coma, and hyperacusis. PCP is very commonly smoked upon application to marijuana. B would be consistent with opioid withdrawal.

20.3 A 26-year-old woman comes to see a psychiatrist because she has been taking showers for 6 to 7 hours every day. She explains, "It all starts when I wake up. I am sure I am covered in germs, and if I don't wash, I will get sick. If I don't wash, I get paralyzed with anxiety. Once I'm in the shower, I have to shower in a particular order. If I mess up, I have to start over, and this takes hours and hours. My skin is cracking and bleeding because I spend so much time in the water." Which of the following disorders does this patient most likely have? A. Obsessive-compulsive disorder B. Obsessive-compulsive personality disorder C. Obsessive-compulsive traits D. Paranoid personality disorder E. Schizoid personality disorder

A. This patient demonstrates the classic obsessions, followed by compulsions, of OCD.

21.1 A 36-year-old businessman who survived a serious car accident 4 months ago complains of "jitteriness" when driving to work and is currently using public transportation because of his anxiety. He has found himself "spacing out" for several minutes at a time and having difficulty concentrating on his job. He has felt "sad" much of the time, has trouble sleeping at night, has lost 4 lb because of a decreased appetite, and admits that his job performance is slipping. Which of the following is the most likely diagnosis? A. Major depressive disorder B. Panic disorder C. Social phobia D. Specific phobia E. Temporal lobe epilepsy

A. This patient exhibits the features of major depression, which commonly occurs with PTSD. In addition, he exhibits two of the three symptom clusters for PTSD, and a clearly traumatic event has preceded his symptoms. It would be unlikely for this patient to suddenly develop an anxiety disorder such as panic disorder, social phobia, or specific phobia. While the "spacing-out" periods are likely to be episodes of dissociation occurring as a result of the trauma, possible neurologic injury should be considered, especially because of his history and the change in his job performance. Also, the patient might be using alcohol to aid in sleeping or to decrease the hypervigilance he has experienced since the accident.

14.4 A 60-year-old man is brought to the emergency room by his wife for "confusion." She reluctantly confides to the staff that he is a "heavy drinker," that he has drunk up to a case of beer almost every day for the past 30 years. Although he has not changed his alcohol intake significantly, over the past year he has eaten less, preferring alcohol to large meals. She has noticed a gradual weight loss as a result. His last drink was earlier this day. Which of the following would be the most likely finding on the mental status examination of this patient? A. Confabulation B. Delusions C. Elevated affect D. Fluctuating consciousness E. Loose associations

A. This patient has a long history of heavy, regular alcohol use and likely malnutrition. A common sequela of this is chronic thiamine deficiency, resulting in Korsakoff syndrome. Korsakoff syndrome is characterized by an anterograde amnesia; this memory impairment is often (poorly) compensated for by the patient's confabulation, or filling in the missing memories with false information.

21.2 The patient in Question 21.1 is referred to a therapist to begin psychotherapy for his PTSD. Which of the following has been shown to be the most efficacious treatment for his condition? A. Cognitive-behavioral therapy B. Dialectical-behavioral therapy C. Hypnotherapy D. Insight-oriented therapy E. Psychoanalysis

A. While different types of psychotherapy have been found to be effective in PTSD, various forms of cognitive-behavioral therapy have been the best studied and most useful. Dialectical-behavioral therapy is a specific therapy developed for the treatment of borderline personality disorder. Neither hypnotherapy nor psychoanalysis has been adequately studied for the treatment of PTSD patients.

18.2 A 17-year-old honors student is brought to the emergency department by his parents. In the last academic quarter, his grades have suddenly dropped, he is irritable with friends and family, he has no energy, he does not go to bed until 1 AM, and he has a poor appetite. He also has auditory hallucinations in which a man's voice tells him that he is a "lazy bastard" and that his family "would be better off with him dead." Which of the following would be the most appropriate initial pharmacologic treatment plan? A. Benzodiazepine B. Antidepressant and antipsychotic medications C. Antidepressant medication D. Antipsychotic medication and a benzodiazepine E. Antidepressant medication and lithium

B. An antidepressant combined with an antipsychotic agent is needed for this patient who has major depression with psychotic symptoms. A benzodiazepine can address neither the depressive nor the psychotic symptoms. Lithium is an augmenting agent that can be considered later if necessary.

7.1 An 80-year-old man undergoes an evaluation for dementia versus depression. His caregiver describes a history of gradually worsening depressed mood and confusion, with poor appetite, weight loss, poor self-care, and irritability. On his cognitive examination, he is alert and oriented to person and place but not to time. His concentration is impaired, and he displays poor short-term memory despite adequate recall. His effort is poor overall, and he often responds to questions stating, "I don't know." Which of the following features is more consistent with a depressive illness than with dementia? A. Poor concentration B. Poor effort during the interview C. Poor self-care D. Poor short-term memory E. Poor appetite

B. Difficulty concentrating, decreased self-care, poor appetite, and short-term memory deficits can be seen in both severe depression and dementia in elderly patients. However, during the cognitive examination, patients with depression usually make little effort but have considerable insight into their difficulties, whereas patients with dementia often make considerable effort but display confabulation and little insight into their mistakes.

29.1 Bulimia differs from anorexia nervosa in which of the following ways? A. Patients with bulimia tend to be low achievers in academics compared to patients with anorexia. B. Patients with bulimia may not have any symptoms until early adulthood while anorexia typically begins in early adolescence. C. Patients with bulimia are less likely to abuse alcohol and have less emotional lability than patients with anorexia. D. Bulimic patients tend to be overweight, whereas anorexic patients are underweight. E. Patients with bulimia are more resistant to receiving help and often must be forced to see a therapist.

B. Patient with both disorders tend to be high achievers but patients with bulimia tend to be less resistant to getting help, have more alcohol abuse, and have more emotional lability than patients with anorexia, who tend to be more emotionally constricted. Bulimia often has a later onset than anorexia.

18.3 Psychotic depression is diagnosed in a 14-year-old boy, and he is treated with an antipsychotic agent, risperidone, and an antidepressant. Three months later, his mood symptoms have resolved, and he is no longer psychotic. Which of the following best describes the next step? A. Both medications should be discontinued via a taper. B. The antipsychotic medication should be discontinued via a taper. C. The antidepressant medication should be discontinued via a taper. D. Both agents should be continued for 6 to 9 months. E. The antipsychotic medication should be stopped immediately.

B. The antipsychotic agent should be discontinued because the psychotic symptoms have abated. The antidepressant should be continued for approximately 6 to 9 months in a patient with the first onset of major depression, and for longer (perhaps indefinitely) for a patient with recurrent depression.

7.3 The patient in Question 7.2 returns after 2 weeks. Despite compliance with nortriptyline, her depressive symptoms have since worsened, with the new onset of suicidal ideation without a plan. Upon further questioning, she admits to feeling that unknown individuals are following her and to hearing auditory hallucinations calling her name and making derogatory comments. Consideration is given to beginning a trial of ECT. Which of the following is the best indication for using ECT in this patient? A. Multiple medical problems B. Psychotic symptoms C. Recurrent episodes D. Suicidal ideation without a plan E. Significant neurovegetative symptoms

B. The most appropriate treatment for a patient with major depressive disorder with psychotic features (whether elderly or not) remains ECT or an antidepressant/antipsychotic combination. Other indications for ECT are when a rapid response is necessary, such as in an imminently suicidal patient, a catatonic patient, or a patient who is not ingesting adequate food or fluid. Neither recurrent episodes nor the presence of neurovegetative symptoms are an indication for ECT.

24.3 A 36-year-old man is referred to a primary care physician for evaluation of his complaints. He is convinced that he has colon cancer despite being told that it is unlikely because of his young age. He occasionally notices blood on the toilet paper and has abdominal cramps when he eats too much. A review of the records demonstrates numerous prior appointments in connection with the same or similar complaints, including repeatedly negative results from tests for occult fecal blood and normal results from colonoscopies. He continues to be worried about dying of cancer and requests another colonoscopy. Which of the following is the most likely diagnosis? A. Body dysmorphic disorder B. Hypochondriasis C. Pain disorder D. Somatization disorder E. Disorder

B. The most likely diagnosis for this man is hypochondriasis. His chief complaint is a concern that he has colon cancer. He remains focused on this illness despite prior evaluations with negative results and reassurance from his physician. Although he has several gastrointestinal symptoms (blood in stools and abdominal cramps), he is probably misinterpreting them. His worry is caused by fears of having colon cancer, not about a distorted body image, pain sensations, or numerous physical symptoms.

4.3 A 52-year-old male executive presents with an onset of depression, early-morning awakening, decreased energy, distractibility, anhedonia, poor appetite, and weight loss for the past 3 months. His symptoms began shortly after he suffered a myocardial infarction. Although he did not experience significant sequelae, he has felt less motivated and fulfilled in his life and work, believing that he is now "vulnerable." As a result, he does not push himself as he used to, and his work output is beginning to decline. A. Bipolar disorder, manic B. Major depression C. Mood disorder due to a general medical condition D. Substance-induced mood disorder E. Adjustment disorder with depressed mood

B. The most likely diagnosis for this man is major depression. He has symptoms typical of the disorder, both depressed mood and neurovegetative symptoms lasting for more than 2 weeks. Although his condition was preceded by a heart attack, it was not a physiologic cause of his depression. Rather, his medical illness (and subsequent feeling of vulnerability) was the stressor that brought on his episode of depression.

2.4 A 27-year-old woman states that for approximately 6 months she has believed that Justin Bieber is in love with her. She insists that he has professed his intentions to marry her through messages in his song lyrics. She has written numerous letters to him and loitered around his home, resulting in several arrests. She is irritated because, although he won't meet with her in person, he often calls her name outside her window when no one else is around. For the past several weeks, she has slept approximately only 2 hours a night but still has enough energy to continuously redecorate her apartment in preparation for her wedding to Mr Bieber. She admits to feeling "on top of the world" because Justin Bieber has chosen her and that she "can't stop talking about it." A. Major depression with psychotic features B. Schizoaffective disorder C. Schizophrenia D. Psychosis secondary to a general medical condition E. Substance-induced psychotic disorder

B. The most likely diagnosis for this woman is schizoaffective disorder. She describes a 6-month history of ideas of reference, delusions, and auditory hallucinations. In addition, she has had clear manic symptoms for the past month, including an elevated mood, a decreased need for sleep, increased energy, increased goal-directed activities, and talkativeness. Although she has symptoms consistent with schizophrenia, she has also had a significant episode of mood disorder during her psychotic illness. Her psychotic symptoms, which preceded and occurred in the absence of mood symptoms, make primary mood disorder (mania) with psychotic features less likely.

24.2 A 26-year-old woman presents to her physician with the chief complaint of, "I have epilepsy." She states that for the past 3 weeks she has had seizures almost daily. She describes the episodes as falling on the ground, followed by shaking her arms and legs uncontrollably. These events last for approximately 10 minutes. She is unable to otherwise move during the time, although she denies any loss of consciousness, bladder, or bowel functions. She has never injured herself during these, but as a result she has been unable to continue her job. She is somewhat bothered as she received a promotion 1 month ago. Which of the following is the most likely diagnosis? A. Body dysmorphic disorder B. Conversion disorder C. Hypochondriasis D. Seizure disorder E. Somatization disorder

B. The most likely diagnosis in this woman is conversion disorder ("pseudo seizures"). Conversion disorder patients present with neurological symptoms (eg, sensory deficit, motor weakness, seizures) that are felt to be unconsciously produced and believed to be caused by a psychological conflict or stressor. It is unlikely a seizure disorder given her retention of consciousness and lack of incontinence or injury. Her focus is not on an imagined defect in appearance, on the fear of having a serious illness caused by misperceived body sensations, or on multiple physical complaints.

13.1 A 17-year-old high school senior is referred to a psychiatrist by his counselor because of academic difficulty. Although he had always been an honors student, this past year his grades have quickly dropped, especially in mathematics. When questioned, he reveals the new onset of "superstitions" involving numbers. When presented with certain numbers, he feels compelled to count forward and then backward to and from that number. He becomes anxious about not completing this task, although he is unable to state a particular consequence. If interrupted, he must begin all over again. He realizes that there is "no good reason" for his behavior, but is unable to stop it. As a result of this, he not only feels "tortured," but he may need to repeat this year in school. He denies any past psychiatric history. He had an appendectomy at age 15 and takes no medications. He does not drink alcohol or use tobacco products or illicit drugs. Which of the following treatments has demonstrated effectiveness in treating this condition? A. Cognitive-behavioral therapy B. Individual and family cognitive-behavioral therapy plus pharmacotherapy C. Pharmacotherapy alone D. Family therapy E. Psychodynamic psychotherapy plus pharmacotherapy

B. This individual suffers from OCD with his obsessions, compulsions, significant anxiety, and interference in his academic functioning. Whereas behavioral therapy and medications such as clomipramine and SSRIs are helpful in treating OCD, there is evidence that a combination of the two provides the greatest efficacy. There is an absence of studies documenting improvement in OCD solely with psychodynamic psychotherapy.

3.2 A 40-year-old woman presents with complaints of not being able to leave her house. For the past 5 years, she has had increasing difficulty traveling far from home. She constantly worries that she will not be able to get help if she "freaks out." In fact, she has had numerous unprovoked episodes of intense fear, associated with shortness of breath, chest pain, diaphoresis, and dizziness, lasting for 20 minutes. She is convinced that if she drives too far from home, she will have an attack and not be able to obtain help. Which of the following is the most likely diagnosis? A. Generalized anxiety disorder B. Panic disorder with agoraphobia C. Panic disorder without agoraphobia D. Social phobia E. Specific phobia

B. This woman most likely has panic disorder with agoraphobia. She experiences recurrent spontaneous panic attacks and between attacks worries about having further attacks. She avoids driving away from her home for fear of being unable to obtain help in the event of an attack.

23.4 As the dementia in the patient in Question 23.1 progresses further, he becomes more agitated and aggressive. Which of the following treatment options would be the most appropriate for his behavior? A. Acetylcholinesterase inhibitor B. High-potency antipsychotic C. Long-acting benzodiazepine D. Low-dose antidepressant E. Low-potency antipsychotic

B. Treatment with a low dose of a high-potency antipsychotic is an appropriate pharmacologic approach to decreasing agitation and aggression in individuals with dementia. Lower-potency antipsychotics should be avoided given their significant anticholinergic and orthostatic side effects. Short-acting benzodiazepines can also be used but may disinhibit patients and worsen their behavior. Long-acting benzodiazepines should be avoided because they can build up in elderly patients and cause oversedation or ataxia. Antidepressants can be useful in treating the depressive and neurovegetative symptoms that are common in dementia, but they would not address the agitation or hostility. Acetylcholinesterase inhibitors can be helpful in improving cognition in the early and mid-stages of Alzheimer disease.

14.1 A 28-year-old man presents with a 12-year history of regular alcohol use. Although he has been able to maintain employment as a truck driver, he often drives when "buzzed" in order to make his deadlines. He has been reprimanded on numerous occasions for failure to perform his job adequately, and this has led to increasing conflict with his wife. He denies any recent increase in drinking or any withdrawal symptoms, but he does admit to not getting "as drunk as I used to" following consumption of the same amount of alcohol. Which of the following factors in his history is the most specific for alcohol dependence? A. Driving while intoxicated B. Marital conflicts C. Not becoming intoxicated as easily D. Occupational problems E. 12-year history of regular alcohol use

C. Although DWI, marital conflicts, and occupational problems are all criteria for alcohol abuse, only not becoming intoxicated as easily as in the past is a criterion for alcohol dependence. Tolerance for alcohol (experiencing either the same effect with an increased amount or a decreased effect with the same amount), withdrawal from alcohol, and an inability to control alcohol use are the characteristics of alcohol dependence.

26.1 A 17-year-old adolescent girl is seen in your office after her friends noticed some strange behavior. The patient reports to you that in addition to some long-term depression issues, she has begun to experience some other disturbing events. She reports that over the last 2 months she has been hearing voices—both at work and at home—of people who she does not think are there. She doesn't recognize these voices. Sometimes they just give a dialogue of what she is doing, but more disturbing to her is when they start saying horrible things about her and tell her to do things she does not want to do. You start her on olanzapine and when she returns in 1 week the voices have been gone entirely for 2 or 3 days. However, she continues to experience severe mood symptoms. Her Hamilton Depression rating scale score places her in the moderate to severe range for depression. What should you do next? A. Inform the patient that these symptoms are the negative symptoms common to the disorder. B. Refer the patient for supportive psychotherapy. C. Treat the patient with fluoxetine (an SSRI). D. Increase the dose of the antipsychotic. E. Add a mood stabilizer to the regimen.

C. Although data are not clear as to the efficacy of administering antidepressants to patients with schizoaffective disorder (and depressive symptoms), the continued presence of depressive symptoms makes this treatment worth trying.

28.3 Atomoxetine is a relatively new drug used for the treatment of ADHD. Which of the following represents the advantage of using atomoxetine over Ritalin? A. Atomoxetine has a shorter half-life. B. Atomoxetine is available in a generic form that is less costly than Ritalin. C. Atomoxetine appears to have less of a potential for abuse than does Ritalin. D. Atomoxetine's effects begin working immediately to reduce symptoms of ADHD. E. Atomoxetine can be taken on an empty stomach.

C. Atomoxetine appears to have a lower potential for abuse than Ritalin. Initial use can produce a feeling of sleepiness and grogginess, and the drug does not appear to exhibit its effects for up to 3 weeks. It is not recommended to take atomoxetine on an empty stomach, as side effects of the drug include nausea and vomiting. Atomoxetine is not currently available in a generic form. One of the advantages of this drug is that it affords 24-hour control of ADHD symptoms (ie, it has a longer half-life than Ritalin).

28.2 A 6-year-old boy with an early, ongoing history of distractibility, hyperactivity, and impulsivity is diagnosed with ADHD. He is treated with methylphenidate. Three weeks later he is brought in and his inattention and hyperactivity are much better. The mother also notes that he has a small bald spot from where he has begun repeatedly rubbing his head. You periodically observe him to suddenly raise his hand to the spot, rub back and forth once, and put his hand down. There is no rash but the area is hairless. The most likely diagnosis is which of the following? A. Alopecia secondary to stimulant B. Drug allergy with contact dermatitis C. Stimulant-induced complex motor tic D. Scabies E. Attention-seeking behavior

C. Atomoxetine will not promote the tics. Methylphenidate and amphetamine salts will promote the tics. Buproprion can stimulate dopamine and worsen tics.

5.2 A 16-year-old girl has been admitted with a 3-week history of sudden irritability, impulsive buying, and disappearing at night with older men. Her need for sleep is decreased; she has flight of ideas and grandiose thoughts about being an advisor to a presidential candidate. Routine admission labs indicate she is pregnant. Which of the following is a statement that should be made to her parents? A. Treatment with an SSRI antidepressant is a reasonable alternative to mood stabilizers in a pregnant girl. B. Given the fact that she is pregnant, she should be kept secluded on an inpatient unit during the first trimester of pregnancy with no medications. C. An atypical antipsychotic may be the best choice for managing both psychotic features and mood disturbance associated with her bipolar disorder, especially during the first trimester. D. Psychotherapy will have little role in the treatment of her bipolar disorder. E. Lithium, divalproex, and carbamazepine are all reasonable first-choice mood stabilizers for this patient.

C. Atypical antipsychotics are good choices for controlling mania in pregnancy given the strong teratogenic effects of most mood stabilizers. SSRIs will only make the mania worse. The patient will need psychotherapy focused on helping her sort out her feelings about the pregnancy.

27.2 A 30-year-old woman is brought to her physician by her brother, who states that the woman has been having numerous psychiatric symptoms. These include hearing voices, seeing "ghosts," and tasting a sour taste even when she has not eaten anything. On mental status examination by the physician, the patient is also noted to have problems with her reality testing and loose associations. Which of the patient's symptoms are most indicative of a psychosis secondary to a general medical condition? A. Auditory hallucinations B. Impairment in reality testing C. Gustatory hallucinations D. Thought disorder (ie, loose associations) E. Visual hallucinations

C. Auditory and visual hallucinations are common in all episodes of psychosis regardless of the cause. Gustatory hallucinations (along with olfactory and tactile hallucinations) can be more common in psychoses caused by a medical illness. It would be somewhat unusual to find problems with reality testing or thought disorders in a patient with psychosis due to a general medical condition, though this is possible. These are much more commonly seen in other psychotic disorders such as schizophrenia.

10.1 A 37-year-old woman is referred to your office for psychotherapy. She is preoccupied with thoughts of her husband leaving her and being left alone with nobody to take care of her. She feels sad and hopeless when alone, lacks self-confidence, and has great difficulty making decisions. You believe she suffers from a personality disorder. In which cluster does the patient's most likely diagnosis belong? A. Cluster A B. Cluster B C. Cluster C D. Cluster D E. Cluster E

C. Cluster C can also be remembered as the "sad" cluster (see Case 6). It includes dependent, avoidant, and obsessive-compulsive personality disorders. The clusters can be remembered by the words "mad" (cluster A—characterized by odd or eccentric behavior), "bad" (cluster B—characterized by dramatic or emotional behavior), and "sad" (cluster C—characterized by anxious or fearful behavior)

25.3 A 39-year-old man with antisocial personality disorder, incarcerated for life after murdering a man, has a multitude of somatic complaints over the course of several years. Yearly physical examinations never show anything physically wrong with him, yet he complains of a variety of aches and pains, neurologic symptoms, and gastrointestinal distress. He does not enjoy the time he spends in the jail's infirmary. Which of the following is the most likely explanation for this patient's complaints? A. He is malingering. B. He has developed a psychotic disorder. C. He has developed a somatization disorder. D. He has an undiagnosed physical illness. E. He has an undiagnosed anxiety disorder.

C. Development of a somatization disorder becomes more common in patients with antisocial personality disorder as they grow older. There is no evidence of secondary gain here (which rules out malingering), nor is there evidence of psychotic thinking. Physical examinations have all been negative (making a physical illness less likely) and the complaints are all around somatic symptoms, making a pure anxiety disorder less likely as well.

15.3 A 25-year-old man with schizotypal personality disorder comes to his psychiatrist with a chief complaint of a depressed mood. He notes that since losing his job as an astrologer, he has been depressed and unable to sleep. He states that although his mood is usually fairly low (4 out of a possible 10), it has lately been a constant 2. The patient also notes problems with concentration and energy level and has experienced several crying spells. He reports he had premonitions that certain foods could heal him, so he has been mixing "magical potions" and eating "magical foods." A mental status examination reveals an oddly dressed man with constricted affect, ideas of reference, unusual beliefs, and some mild paranoia. Which of the following medications is most likely to be helpful to this patient? A. Zolpidem (Ambien) for insomnia B. Divalproex sodium for mood disturbance C. Escitalopram for depressive symptoms D. Risperidone for paranoia E. Ziprasidone for ideas of reference

C. Escitalopram is a selective serotonin reuptake inhibitor (SSRI) useful in the treatment of depression. Patients with schizotypal personality disorder who have either a depressive component to their illness or a secondary superimposed major depression (as is the case with this patient) should be treated with antidepressants. Ziprasidone (Geodon) and risperidone (Risperdal) are atypical antipsychotics that would be effective if the patient was having a transient psychotic episode, but he is not. Divalproex sodium is an antiepileptic agent used as a mood stabilizer to treat conditions such as mania.

19.3 Which of the following treatments is best employed to treat the comorbid depressive symptoms of an adolescent with CD? A. Multisystemic therapy B. Attendance in group therapy C. An antidepressant medication D. Treatment of the family to address the underlying reasons for the depression E. Helping the adolescent change schools

C. If the criteria for a comorbid condition are met, this disorder should be the first target of psychopharmacologic intervention.

12.1 A 62-year-old man presents to his primary care physician at the insistence of his wife. The man states that there is nothing wrong with him, but that he has not been sleeping nearly as much as he used to, often needing less than 2 hours of sleep a night. His wife notes that he has been exceedingly irritable, has been charging excessive amounts of money on their credit cards, and has been talking about running in a marathon, although he has never expressed such an interest before. The patient has no previous psychiatric or medical history. He denies the use of drugs or alcohol. Which of the following courses of action should the physician take first? A. Admit to the hospital. B. Order a urinalysis for drugs of abuse. C. Perform a complete physical examination. D. Start a mood-stabilizing drug. E. Start an antipsychotic drug.

C. In a patient of this age, with no previous psychiatric or medical history, general medical conditions responsible for this new-onset behavior should be ruled out before an episode of bipolar mania is considered. While admission to the hospital might well be necessary to control the destructive behavior, making the diagnosis first is the primary concern. Both a mood stabilizer and/or an antipsychotic may well also be necessary, but again, if the mood problem is secondary to a general medical condition, treating it may resolve the psychiatric symptoms by itself.

9.1 A 39-year-old man presents to the emergency room at the behest of his girlfriend, who reports that he has not slept in 3 days. The patient speaks extremely rapidly, and switches topic so frequently as to be incomprehensible. His affect is happy and elevated, but he quickly snaps and becomes belligerent when he is accidentally bumped by a nurse. Which of the following symptoms would most likely distinguish this patient's presentation from PCP intoxication? A. Disorganized thoughts B. Hostile or violent behavior C. Nystagmus D. Hallucinations E. Pressured speech

C. Individuals with mania and PCP intoxication can have hallucinations, display hostility, and have disordered thoughts; nystagmus is commonly associated with PCP use but not with mania.

20.1 A 24-year-old woman is called into the head office of the agency where she works and told that her chronic lateness in completing her assignments will result in her dismissal if she does not change her behavior. The patient really loves her job, and the news comes as a major blow. That night at home, she tells her boyfriend in great detail about each and every step of the meeting and spends the entire night thinking about her job. The boyfriend tells her that she does not "look" particularly upset. Which of the following defense mechanisms is being used by this woman? A. Undoing B. Displacement C. Intellectualization D. Rationalization E. Splitting

C. Intellectualization is a defense mechanism by which an individual deals with emotional conflict or stressors with an excessive use of abstract thinking to control or minimize disturbing feelings. Because the stressors have been successfully defended against in this instance, the patient does not appear particularly distressed.

9.2 A 15-year-old boy is brought to the emergency department by the police due to violent, psychotic behavior. Phencyclidine intoxication is confirmed via urine toxicology. Which of the following treatment interventions is associated with the lowest risk of adverse complications? A. Low-potency, traditional antipsychotic agents to treat hallucinations B. Ammonium chloride to acidify the urine and increase clearance of the drug C. Benzodiazepines for agitation D. Gastric lavage to remove unabsorbed, excess drug E. Full-leather restraints to prevent harm to self or others

C. Low-potency traditional antipsychotics may worsen intoxication syndrome via anticholinergic side effects. Acidification of the urine has been found to be ineffective, and increases the risk of acute tubular necrosis due to myoglobinuria in the rhabdomyolysis. Gastric lavage is contraindicated due to risk of emesis and aspiration, while restraints may lead to muscle breakdown.

18.1 A 10-year-old girl is brought in for treatment by her father following the death of her mother 6 weeks previously because of an unexpected heart attack. The father is worried because the child is not sleeping well, has lost 7 lb because of a decreased appetite, seems to be tired much of the time, and is preoccupied with memories of her mother. He notes that she cannot concentrate on her usual favorite television shows and has lost interest in many of her previous social activities. The patient reports that she deeply misses her mother, but she also smiles in recalling many pleasant memories of their life together. Which of the following is the most likely diagnosis? A. Adjustment disorder with depressed mood B. Major depression C. Normal bereavement D. Sleep disorder E. Dysthymic disorder

C. Normal bereavement. This patient's symptoms fulfill the criteria for major depression, but she is in the very early stages of bereavement when such behavior is considered normal. Her sleep problems are part of bereavement. If her symptoms do not diminish in the subsequent 4 months, the clinician should make a reassessment for major depression.

6.2 A patient with schizoid personality disorder comes to his primary care physician with chief complaints of polyuria and polydipsia. He is found to have insulin-dependent diabetes. Which of the following interventions by the physician is likely to be most well received by this patient? A. Asking the patient to bring in a relative so that he can describe the treatment regimen to both of them at the same time. B. Referring the patient to a therapist so that he can talk about the difficult nature of the diagnosis. C. Giving the patient detailed written information about the disease and telling him that the physician will be available to answer any questions. D. Referring the patient to a group that helps its members learn about diabetes and to better deal with their illness. E. Scheduling frequent appointments with the patient so that all the treatment details can be explained on a one-to-one basis.

C. Patients with schizoid personality disorder generally prefer to keep social interaction to a minimum. They do well with technical information.

25.2 A 16-year-old adolescent girl is incarcerated in a juvenile detention facility. She is currently charged with theft, apparently to support her and her boyfriend's drug habit. She has had multiple involvements with child and family services for running away from home, where she apparently had been sexually abused by her mother's boyfriend. She has a diagnosis of posttraumatic stress disorder (PTSD). Prior to the onset of the abuse, she was doing extremely well in school, in an accelerated program. Which of the following factors speaks most strongly against a diagnosis of antisocial personality disorder? A. Her concurrent diagnosis of PTSD B. Her gender C. Her age D. Antisocial acts committed to support a drug habit E. Apparent high intelligence

C. Personality disorders cannot be diagnosed before 18 years of age. Antisocial personality disorder should not be overlooked in females, even though it is much more common in males. Antisocial actions committed solely during psychotic or manic episodes, or to support a drug habit, would not support a diagnosis of antisocial personality disorder.

27.3 A 25-year-old man sustained head trauma as a result of his car hitting a tree. After all other causes have been eliminated, it is determined that he has developed psychotic hallucinations because of the head trauma. Which of the following medications is most likely to be helpful in treating this patient's psychosis? A. Lithium B. Valproic acid C. Risperidone D. Valium E. Sertraline

C. Psychoses caused by general medical conditions usually respond to antipsychotic medications. There is no indication for the use of drugs used to treat mood disorders (lithium, valproic acid, or sertraline) or benzodiazepines.

22.3 The patient in Questions 22.1 and 22.2 returns to your office for a medication check. Sertraline was started 4 months ago. The dose has been increased twice and the patient has been taking 200 mg for 2 months. She does not feel the medication has provided any relief to her symptoms, but she has tolerated the medication well. What would be the next step for this patient? A. Augment by adding another medication. B. Discontinue and begin a tricyclic antidepressant (TCA). C. Discontinue and begin another selective serotonin reuptake inhibitor (SSRI). D. Give the drug more time. E. Increase the dose.

C. SSRIs are generally tolerated better by patients and are often beneficial in treating dysthymia. Two 200 mg of sertraline is the maximum dose and 6 to 10 weeks is considered an adequate trial, so another SSRI should be tried before switching to another class. Augmentation is usually done when the patient has a partial response to an initial medication.

3.4 A 33-year-old man has the chief complaint of "I'm going to have a heart attack like my father." He explains that his father died of a myocardial infarction at 45 years of age. He is convinced that he is experiencing anginal attacks consisting of nervousness, sweating, palpitations, flushing, and numbness in his hands lasting for several minutes. He is anxious about having these symptoms and, despite negative results from a cardiology workup, remains certain that he will suffer a heart attack. His behavior and lifestyle have not been otherwise affected. Which of the following is the most likely diagnosis? A. Generalized anxiety disorder B. Panic disorder with agoraphobia C. Panic disorder without agoraphobia D. Social phobia E. Specific phobia

C. The most likely diagnosis for this man is panic disorder without agoraphobia. He displays characteristic features of panic attacks, such as recurrent episodes of anxiety associated with physical symptoms. These episodes are spontaneous, and he worries about the consequences of having an additional attack, namely, a myocardial infarction. However, he does not avoid places, and his behavior is otherwise unaffected.

2.2 A 46-year-old man presents with a long-standing belief that his thoughts are being taken from his head and used to create a blockbuster movie. He is certain that the government is involved because they often communicate with him through a microchip they have implanted in his brain. Although he feels frustrated at being taken advantage of, he denies any significant depressive symptoms and is often able to enjoy playing cards with his peers at the group home. A. Major depression with psychotic features B. Schizoaffective disorder C. Schizophrenia D. Psychosis secondary to a general medical condition E. Substance-induced psychotic disorder

C. The most likely diagnosis for this man is schizophrenia. He has been suffering from psychotic symptoms including delusions and auditory hallucinations for more than 6 months. Although he can have brief periods of depressed mood, he does not have a history of major mood disorder.

4.4 An 80-year-old woman without a psychiatric history is examined after a left-sided cerebral vascular accident that has left her paralyzed on her right side. Since her stroke, she complains of an absence of pleasure in anything that she formerly enjoyed. She describes frequent crying spells, increased sleeping, a decreased appetite with weight loss, and feelings of hopelessness and helplessness. A. Bipolar disorder, manic B. Major depression C. Mood disorder due to a general medical condition D. Substance-induced mood disorder E. Adjustment disorder with depressed mood

C. The most likely diagnosis in this case is mood disorder caused by a general medical condition, namely, a cerebral vascular accident. The patient has obvious symptoms of a depressive illness, including anhedonia. These symptoms also have a clear temporal relationship to her stroke, which has left her with significant motor deficits. Cerebral vascular events are well known to result in depression.

4.2 A 39-year-old woman presents with 1 month of a gradually worsening depressed mood, with increased sleeping, low energy, and difficulty concentrating, but no appetite or weight changes. Her medical history is significant for multiple sclerosis, but she is currently not taking any medication. Her mental status examination is notable for psychomotor slowing and a depressed and blunted affect. Her physical examination demonstrates several different sensory and motor deficits. A. Bipolar disorder, manic B. Major depression C. Mood disorder due to a general medical condition D. Substance-induced mood disorder E. Adjustment disorder with depressed mood

C. The most likely diagnosis in this case is mood disorder due to a general medical condition, namely, multiple sclerosis. Although this woman displays the characteristic symptoms of an episode of major depression (depressed mood, increased sleeping, low energy), she does not exhibit the appetite or weight changes commonly seen in this illness. Steroids can often cause mood symptoms such as depression or mania, but she is currently not taking any medication. The results of her physical examination are also consistent with a flare-up of her multiple sclerosis and demonstrate a temporal relationship to her depression. The central nervous system white matter lesions seen on imaging are known to be associated with a depressive state.

15.2 Which of the following features must be present in a patient's history for schizotypal personality disorder to be diagnosed? A. The patient must have a history of active substance use. B. The patient must have a history of auditory hallucinations. C. The patient must have a history of cognitive and perceptual distortions. D. The patient must have a history of intense, short-lived friendships. E. The patient must have a history of suicidal ideation.

C. The odd quality in which these patients perceive and think about the world is one of the diagnostic criteria for schizotypal personality disorder. Answers A, B, and E are symptoms of a variety of axis I disorders, but do not point to any particular personality disorder. Answer D is a characteristic seen in borderline personality disorder patients, although it is not a diagnostic criterion per se.

19.1 A 14-year-old girl's family brings her to treatment against her wishes. The family reports she has a very short temper, argues with them and teachers at school daily, often refuses requests at both settings, and usually blames her teachers or her parents for the behaviors she is showing. She denies being depressed, anxious, or any psychotic symptoms. With this information, what might be the best diagnosis? A. Antisocial personality disorder B. Borderline personality disorder C. Oppositional defiant disorder D. Conduct disorder E. Posttraumatic stress disorder

C. The patient does not yet meet criteria for conduct disorder, which might include additional signs such as aggression to people or animals, destruction of property, theft, or significant rule violation. Both conditions cause impairment in social, family, and academic functioning.

11.4 A 26-year-old woman with a history of major depressive disorder and generalized anxiety disorder presents to the emergency department with her husband complaining of anxiety, shortness of breath, palpitations, and "feeling like I am going to die." Her symptoms began earlier in the day, but over the past 2 hours have gotten significantly worse. She has no significant past medical history, smokes half a pack of cigarettes per day, and her only medications are oral contraceptives and venlafaxine. On examination, she speaks in short sentences and appears diaphoretic. Her pulse rate is 160 beats per minute, blood pressure 104/64 mm Hg, and respiratory rate is 32 breaths per minute. An ECG shows sinus tachycardia. What is the most appropriate next step? A. Adminster lorazepam. B. Start buspirone. C. Order a high-resolution CT of the chest. D. Tell her she is having a panic attack and would benefit from CBT. E. Inquire about current social stressors.

C. This 26-year-old woman is presenting with classic signs of a pulmonary embolus (tachycardia, hypotension, and tachypnea). Additionally, her use of cigarettes and oral contraceptives places her at greater risk for PE than the general population. Although she has a psychiatric history, dismissing her complaints as a panic attack would be negligent. Lorazepam will not aid in diagnosis and may result in clinical deterioration. Inquiring about social stressors is not appropriate in the setting of hemodynamic instability. A high-resolution CT of the chest is indicated.

27.4 A 24-year-old woman with no past medical or psychiatric history presents to her primary care doctor in tears. Over the past 3 months she has been experiencing intermittent myalgias and arthralgias, joint swelling, rashes, fatigue, chest pains, and feels that she is more sensitive to the sun than previously noted. She came in today on an emergency basis because she is convinced her neighbors have been poisoning her. She is accompanied by her boyfriend who states that she has never had thoughts like this before. Despite all of her somatic symptoms, she has been functioning at home and work for the past several months. There is no history of substance abuse. After a physical examination, what is the next step in management? A. Prescribe citalopram. B. Prescribe olanzapine. C. Order a CBC and ANA. D. Refer the patient for counseling. E. Reassure the patient that no one is trying to poison her.

C. This patient is presenting with classic symptoms of lupus. An appropriate medical workup is indicated prior to attributing her symptoms to psychiatric causes. Generally speaking, answers that including "referring" to someone else are incorrect.

7.4 You are consulted to evaluate an 84-year-old, widowed woman on the medical ward. She has a prior history of major depressive disorder, recurrent, and she was admitted for a syncopal episode. She was found to be extremely malnourished, hypotensive, and in acute renal failure. She has not been taking her antidepressant for many months. She describes having all the neurovegetative symptoms of depression and to not eating or drinking for weeks. When questioned about this, she admits to purposely "starving myself" as she believes that God is punishing her for directly causing the terrorist attacks of September 11, 2001. In fact, she has been "following God's instructions," which tell her to kill herself in atonement. After medical stabilization, which of the following treatments would be the most appropriate for this patient? A. Antidepressant only B. Antipsychotic only C. Electroconvulsive therapy only D. Psychotherapy only E. Psychotherapy plus antidepressant

C. This woman's history and presentation are most consistent with major depression, severe, with psychotic features. The treatment of choice for this illness is either ECT or an antidepressant/antipsychotic combination. Given the medical urgency in this case and that medication would take several weeks for significant efficacy, ECT would be preferable. Neither antidepressants nor antipsychotics alone would be as efficacious in an episode of psychotic depression, and psychotherapy with or without medication would not be appropriate in someone with a severe depression as in this case.

16.1 A 50-year-old homeless veteran is brought to the emergency department by the police for disruptive behavior. On mental status examination, he is clearly euphoric but also psychomotor agitated and somewhat paranoid; he says he "feels fantastic" but is wary of answering any questions, becoming irritated quickly. On physical examination, the patient exhibits a moderately elevated blood pressure and pulse rate. He is most likely intoxicated with which of the following substances? A. Alcohol B. Barbiturates C. Benzodiazepines D. Cocaine E. Opiates

D. Cocaine intoxication (or other stimulants like amphetamine) can present with euphoria, irritability, anxiety, psychotic symptoms such as paranoia, as well as with elevated vital signs. Conversely, intoxication with alcohol, barbiturates, benzodiazepines, and opiates generally causes depression, somnolence, and depressed vital signs.

7.2 A 76-year-old woman with a history of major depressive disorder, recurrent, presents to her family practice physician with a history of 4 months of increasing depression, associated with terminal insomnia, decreased appetite, 15 lb weight loss, fatigue, difficulty concentrating, and feelings of helplessness. She denies suicidal ideation, hallucinations, or delusions. She feels these symptoms are similar to those she had in the past. Her last episode was 30 years ago, when she was successfully treated with nortriptyline, but she has not taken any psychotropics since that time. She has hypertension, diabetes, and hyperlipidemia, but she has no other physical complaints. Physical examination and laboratory studies are unremarkable. After a great deal of discussion, she agrees to restart nortriptyline. When compared with the treatment of her last episode of major depression, which of the following statements is most accurate regarding medication management of this episode? A. She will have fewer side effects. B. She will require higher doses. C. She will require lower blood levels. D. There is the potential for more drug-drug interactions. E. There will be a higher chance of treatment resistance.

D. Geriatric patients are often taking multiple medications and therefore have a higher likelihood of being affected by drug-drug interactions. They are also more likely to experience significant side effects from medications. Although the drug levels required to achieve efficacy are similar to those required in younger patients, because of the decreased clearance and metabolism seen in older patients, lower doses are required to reach the same levels. Efficacy of antidepressants remains similar in elderly patients.

12.4 A 33-year-old woman with bipolar disorder is 22 weeks pregnant. She has been taking valproic acid for her symptoms. Which of the following is the most likely abnormality that might be found on an ultrasound examination, due to the effects of the mood stabilizer? A. Fetal abdominal wall defect B. Fetal microcephaly C. Fetal renal dysplasia D. Fetal spina bifida E. Fetal tetralogy of Fallot

D. Maternal use of valproic acid is associated with a 1% to 2% risk of fetal neural tube defects such as spina bifida.

5.1 Which of the following medications would be the best choice for treating nonpsychotic mania in a 10-year-old boy? A. Isotretinoin (Accutane) B. Beclomethasone C. Lithium D. Divalproex E. Risperidone

D. Mood stabilizers are used to treat bipolar disorder. Lithium is the only medication that has received FDA approval for the treatment of bipolar disorder in youth aged more than 12 years, but divalproex has a longer safety profile in young individuals given its long history of use for seizures.

23.2 Given the loss of a spouse suffered by the patient in Question 23.1, a depressive disorder (pseudodementia) as a cause of his memory problems is considered in his differential diagnosis. How would this patient be predicted to perform on cognitive testing if he has a dementia rather than a depressive illness? A. Reduced effort with poor insight B. Reduced effort with good insight C. No effort with poor insight D. Better effort with poor insight E. Better effort with good insight

D. On cognitive testing, patients with dementia generally put forth considerable effort, display poor insight into their deficits, and minimize their problems. In contrast, those with depressive disorders often are apathetic, make little effort, but complain about their memory problems.

8.1 Which symptom is more likely to be found in a patient with social phobia-related panic attacks than in a patient with a primary panic disorder? A. Paresthesias B. Derealization C. Shortness of breath D. Anticipation of an upcoming event E. Palpitations

D. Panic attacks can occur in both patients with social phobia and patients with panic disorder. In patients with social phobia, however, these panic attacks occur linked to specific social situations and can be expected to occur there. In panic disorder, however, the attacks may occur at any time.

11.3 A 45-year-old woman with alcohol dependence in full remission is referred to your practice. She has a history of GAD and was recently prescribed a benzodiazepine by her internist. Which of the following is the most appropriate treatment for her? A. Continue the benzodiazepine and an SSRI. B. Buspirone monotherapy. C. Continue benzodiazepine. D. Selective serotonin reuptake inhibitor monotherapy. E. Switch to a long-acting benzodiazepine.

D. SSRIs are first-line treatment for GAD. Potentially addictive medications such as benzodiazepines should be avoided in patients with a history of substance dependence. Buspirone is not as effective in patients already exposed to benzodiazepines.

24.4 Which of the following strategies by the primary care doctor would be the most effective in treating the patient in Question 24.3? A. Antianxiety medication B. Extensive medical workups to provide reassurance C. Referral for psychotherapy D. Regularly scheduled appointments with reassurance E. An antipsychotic medication

D. The most effective strategy for treating individuals with hypochondriasis is to schedule regular appointments. In this way, any physical complaints are addressed, and reassurance is provided, albeit temporarily. This approach also minimizes both doctor shopping and unnecessary testing. Treatment with an antianxiety (or antidepressant) agent is not helpful in hypochondriasis, unless a comorbid anxiety (or depressive) disorder is present. Because individuals with this disorder are fearful of having a medical illness, they usually resist seeing a psychiatrist.

3.1 A 28-year-old man describes a persistent fear of speaking in public. Although he does not have difficulty with one-on-one situations, when giving a lecture he becomes extremely anxious, worrying that he will be humiliated. He relates one episode in which he was forced to speak at the last minute, which resulted in his experiencing panic, shaking, abdominal cramps, and a fear that he would defecate on himself. Because of this problem, he has been held back from promotion at his place of business. Which of the following is the most likely diagnosis? A. Generalized anxiety disorder B. Panic disorder with agoraphobia C. Panic disorder without agoraphobia D. Social phobia E. Specific phobia

D. The most likely diagnosis for this man is social phobia. Although he suffers from panic attacks, they are not unprovoked as in panic disorder because they occur in response to public speaking. His fear is not of having further attacks but rather of being embarrassed or humiliated. For generalized anxiety disorder the anxiety and worry is not confined to being embarrassed as in social phobia, but rather involves excessive anxiety and worry about a number of events or activities. Specific phobia is characterized by an excessive fear that is cued by the presence or anticipation of a specific object or situation (eg, flying, animals) rather than social or performance situations.

4.5 A 36-year-old man with a past history of a major depressive episode is brought into the emergency room by the police after stopping traffic on the highway proclaiming that he is "the Messiah." His wife is contacted who states that he has been walking throughout the house all night for the last 4 nights, talking "nonstop," and starting many home repair projects that remain unfinished. She believes that he is taking sertraline for his depression and propranolol for high blood pressure. His blood alcohol level is less than 10, and his urine toxicology screen is negative. A. Bipolar disorder, manic B. Major depression C. Mood disorder due to a general medical condition D. Substance-induced mood disorder E. Adjustment disorder with depressed mood

D. The most likely diagnosis for this man is substance-induced mood disorder, in other words mania caused by antidepressant treatment. He has classic symptoms and signs of mania, such as decreased need for sleep, talkativeness, increased activity, risky behavior, and delusions of grandeur. He also has a history of a major depressive episode and has been taking an antidepressant which likely has caused a switch into his current manic episode. According to the DSM-IV-TR, manic episodes caused by antidepressant treatment are considered substance induced and do not count toward a diagnosis of bipolar disorder. Although he is taking a beta-blocker for hypertension, this would be likely to cause a depressive episode rather than a manic episode.

20.4 For the patient described in Question 20.3, which of the following treatments might best be used by the psychiatrist? A. Lithium B. Interpersonal psychotherapy C. Buspirone D. Cognitive-behavioral therapy (CBT)-evoked response prevention E. Parent assertiveness training

D. The standard pharmacologic approach to the treatment of OCD is to prescribe a selective serotonin reuptake inhibitor (SSRI) or clomipramine, although these are not answer choices. The best psychotherapeutic choice would involve gradually exposing the patient to the anxiety-provoking circumstance and teaching her how to manage that anxiety through CBT techniques.

10.2 You are consulted to evaluate a 45-year-old, married woman who was admitted to the surgical service 2 days ago for an appendectomy. The procedure went well, but she was found to be tearful, stating, "I wish I were dead." On obtaining further history, she is quite cooperative and talkative. She is questioned about her earlier comments, and she states that she "wanted attention, I guess." She is upset that her husband is not with her in the hospital; she has "never been away from him" for this long since they started dating when the patient was 16 years old. She feels helpless and is having a difficult time being active in her care. She feels overwhelmed regarding her postsurgical and discharge instructions, and the nursing staff has become frustrated with her constant "need for reassurance." Although at times she is tearful during the interview, she denies prior or recent pervasive depressive or neurovegetative symptoms and is not actively suicidal. Which of the following is the most appropriate approach to this patient? A. Encourage her to learn more about her surgery and become more proactive in her care. B. Persuade her to become less dependent on her husband. C. Persuade her husband to come spend more time with her at the hospital. D. Spend regular, short periods of time with her to discuss discharge planning and aftercare. E. Transfer her to the psychiatric unit.

D. This patient displays characteristics consistent with dependent personality disorder. The most effective approach in dealing with a patient with this disorder is to respect her need for attachment and schedule limited but regular appointments with her. Individuals with this illness wish to be taken care of, and therefore will not be proactive in their care; in circumstances such as these, it is helpful for the physician to be more active. Encouraging the patient to be less dependent in her primary relationship is not only not helpful, but can be damaging as she can feel rejected and become more regressed, upset, and helpless. Encouraging the husband to spend more time at the hospital only continues the cycle of "being taken care of" and does not facilitate the patient's involvement in postoperative care.

12.2 An 18-year-old man is brought to the emergency department by his friends after he started a fight with one of them and subsequently was knocked unconscious. On awakening, the patient states that he was fighting for the "freedom of the world" and that he was told by "the voices in his head" that his friends held the key to winning the battle. The patient is irritable and restless and paces around in the emergency department. He is unable to sit still for the interview and prefers to stand up, keeping his back to the door as he speaks with the physician. The patient's friends state that he has been withdrawing from them and his schoolwork over the past 18 months and that they think he has become "odd." They report that his mood has become irritable only over the past several days. They say that he hoards random scraps of paper in his room and that his grades have dropped dramatically over the past 6 months. The results of a toxicology screening are negative, as are those of a physical examination, although the examination was limited because of poor cooperation on the part of the patient. Which of the following is the most likely diagnosis? A. Bipolar disorder, manic B. Mood disorder secondary to a general medical condition C. Schizoaffective disorder D. Schizophrenia E. Substance-induced mood disorder

D. This patient is currently experiencing a frank episode of psychosis, which appears to most recently have escalated with the appearance of paranoia (standing with his back to the door as he speaks with the physician) and extreme irritability, which could be confused with manic symptoms. However, he has a history of a disturbance in premorbid functioning that predates the irritability by 18 months. He is also in the prime age range for new-onset schizophrenia, making this the more likely diagnosis.

22.1 Which of the following characteristics is more consistent with dysthymic disorder versus major depression? A. Episodic course. B. Numerous neurovegetative symptoms. C. Presence of psychotic symptoms. D. Severe impairment in functioning. E. Symptoms are of a low intensity.

E. Although the distinction between dysthymic disorder and major depressive disorder can sometimes be challenging (especially if the major depressive illness is chronic and/or recurrent), patients with dysthymic disorder tend to have an earlier onset of lower-intensity symptoms, a more chronic course, fewer neurovegetative symptoms, lack of psychosis, and less severe psychosocial or occupational impairment when compared to individuals with major depression.

23.1 A 75-year-old man is brought in by his daughter to a psychiatrist for an evaluation. He has become increasingly forgetful over the past year, missing engagements with his children and grandchildren. He is also unable to remember directions, resulting in his becoming lost when driving alone. He has no psychiatric history, although his wife died 14 months ago. His medical history is significant for poorly controlled hypertension. Which of the following additional features is necessary in order to accurately diagnose dementia? A. Agitation B. Fluctuation in consciousness C. Radiographic findings D. Hallucinations E. Another cognitive deficit

E. For a diagnosis of dementia, one or more additional cognitive deficits must be present in addition to memory impairment. They can include aphasia, apraxia, agnosia, or a disturbance in executive functioning. Individuals with vascular dementia can demonstrate findings on a CT or MRI scan, such as lacunar infarcts or evidence of past strokes. Patients with Alzheimer disease may demonstrate generalized cortical atrophy and ventricular enlargement. Individuals with dementia usually remain alert, whereas those with delirium display a fluctuation in consciousness. While psychotic symptoms such as delusions and hallucinations, as well as agitation, can be seen, they are neither specific to nor necessary for the diagnosis of dementia.

27.1 A 47-year-old man with hemophilia presents to his physician with new-onset symptoms including hallucinations (he hears his dead grandmother calling his name). He has no prior psychiatric history. He realizes the symptoms are odd, but he is not overly disturbed by them. He is diagnosed with HIV/AIDS. Which of the following management strategies should be tried first? A. Begin the patient on a low-potency typical antipsychotic. B. Begin the patient on an atypical antipsychotic. C. Admit the patient to the psychiatric unit for stabilization and treatment. D. Suggest the patient begin psychodynamic psychotherapy as soon as possible. E. CT of the head.

E. Psychosis due to a general medical condition first and foremost should be treated by identifying and vigorously treating the underlying medical condition. In this immunosuppressed patient with new-onset psychosis, CT of the head is indicated. Since this patient's psychotic symptoms do not bother him tremendously, and he is not behaving in any way that could be dangerous to himself or others, treatment with an antipsychotic is not necessary first line. If such a treatment became necessary, then treatment with a high-potency atypical antipsychotic medication should be tried next. Psychodynamic psychotherapy would not be useful in this scenario.

16.3 The patient in Questions 16.1 and 16.2 is subsequently admitted to a detoxification unit in the hospital. After his euphoria and paranoia resolve, he is able to give a more complete history. He describes a 5-year history of almost daily crack cocaine use, with no periods of sobriety lasting for greater than 7 to 10 days—during these periods, he felt "depressed," with an increased appetite, disrupted sleep, difficulty concentrating, and fatigue. He denies alcohol or other drug use, and his psychiatric review of systems is otherwise negative. Which of the following would be the most appropriate initial treatment for this patient? A. Antidepressant B. Dopamine antagonist C. Dopamine agonist D. Mood stabilizer E. Narcotics Anonymous

E. The most beneficial approach to cocaine addiction without additional psychopathology is to stress abstinence and relapse prevention. Narcotics Anonymous meetings are easily accessible, approachable, and provide ongoing group and individual support. No medications have been consistently proven to prevent cocaine cravings or relapse. While the patient admits to depressive symptoms when abstinent from cocaine, these symptoms are quite common (and self-limiting) during withdrawal. In addition, the time period is not typical of a major depressive disorder. After 4 to 6 weeks of sobriety, if depressive symptoms are still present, he should be reevaluated with consideration to begin an antidepressant.

1.2 A 44-year-old woman comes to your office for a follow-up visit. She recently received a diagnosis of major depressive disorder and began treatment with citalopram (an SSRI) 6 weeks ago. She claims to feel "happy again," without further depression, crying spells, or insomnia. Her appetite has improved, and she has been able to focus at work and enjoy time with her family. Although she experienced occasional headaches and loose stools at the beginning of her treatment, she no longer complains of any side effects. Which of the following is the most appropriate next step in her treatment? A. Consider a different class of antidepressants. B. Discontinue the citalopram. C. Increase the dose of citalopram. D. Lower the dose of citalopram. E. Maintain the current dose of citalopram.

E. The proper strategy in the management of an episode of major depression that has recently remitted is to continue treatment at the same dose if it can be tolerated. Early discontinuation of medication can lead to an early relapse. A general rule of thumb is, "The dose that got you better will keep you well." A reasonable duration for continuing the medication is 6 to 9 months.


संबंधित स्टडी सेट्स

**chem unit 1 practice problems v2

View Set

NYSTCE CST Multisubject Part 1 (241)

View Set

Ch. 37: Child with a Communicable Disease

View Set

Unit 12 Interest Groups Key Terms and Quiz

View Set

Chapter 52: Care of the Patient with a Sensory Disorder

View Set